Вы находитесь на странице: 1из 123

Contents

A) MSK........................................................................................................... 2
B) GIT........................................................................................................... 19
C) Genitourinary............................................................................................ 33
D) Ultrasound................................................................................................ 46
E) Interventional............................................................................................ 66
F) Chest......................................................................................................... 68
G) CNS.......................................................................................................... 81
H) Head and Neck......................................................................................... 91
I) Cardiovascular........................................................................................... 95
J) Breast....................................................................................................... 103
K) Pediatrics................................................................................................ 110
L) Contrast and safety................................................................................. 121
M) Isotope................................................................................................... 128
................................................131

images

-1-

A) MSK
1- Plane x-ray for elbow anatomy pointing to the olecrenon fossa and
trochlea

2- Articular cartiage is supplied by:


a- nutrient artery
b- synovial fluid
c- others
3- MCQ about 4 mri shoulder Image sequences

T1

T2

T2*

STIR

3- MRI: shoulder I think gradient


4- Shoulder MRI: asking about MRI sequence: ( T1 fat sat with intra
articular Gd)
5- Four images of knee MRI: what are the
sequences:
a- axial PD, axial T2, sag T1 and saittal T2
(this was my answer)
b- other 3 choices

-2-

6- MCQ montagia fracture most common feature


a- upper of shaft of ulna
b- diastasis of distal radio ulnar joint
c- distal radioulnar ligament disruption
d- full down on outstretched hand I shows this.
N.B The Monteggia fracture is # of the proximal of the ulnar shaft with
radial head dislocation.

Monteggia fracture: Fall on an outstretched hand with the forearm in


excessive pronation (hyper-pronation injury). The Ulna fractures in the
proximal of the shaft due to extreme dislocation. Depending on the impact
and forces applied in each direction, degree of energy absorption determines
pattern, involvement of the radial head and whether or not open soft tissue
occurs.
Smith fracture: distal radius # with ventral angulation commonly associated
with ulnar styloid # = reverse Colle's fracture.
Colle's fracture: distal radius # with dorsal angulation commonly associated
with ulnar styloid # = dinner fork configuration of lateral X-ray of wrist.
Bennett's fracture: # of the proximal end of the 1st metacarpal which extends
into the 1st carpo-metacarpal joint.
Salter-Harris II fracture: # through the growth plate and metaphysis.
fractures
Lisfrance, and Gamekeeper fractures
7- Bennett`s fracture.
# of proximal of 1st metacarbal bone extend into
1st metacarbophalngeal joint
8- X-ray salter harris type II
Salter-Harris II fracture: # through the growth
plate & metaphysis
-3-

9- Fractures
a- Monteggia fracture: ulnar shaft, associated with cortical affection.
b- Smith fracture: distal radius fracture with dorsal angulation.
c- Colle's fracture: distal radius fracture with disruption radioulnal joint
d- Bennett's fracture: a fracture of the proximal end of the 1 st metacarpal
without extention into the 1st carpo-metacarpal joint. I know this is wrong
because he mentioned (without extention into 1 st carpo-metacarpal joint)
but other choices were also confusing.
10-Fracture neck femur
a- Always valgus deformity
b- Intracapsular type different in ttt than extracapsular type
11- MCQ regarding pathological fracture
a- Pathological fracture occurs in normal bone with trivial trauma
b- Stress fracture occur in normal bone due to repeated trauma (
)
c- Insuffeciency fracture occur in abnormal bone with major trauma
12-Which ligament is liable to tear with valgus force to the elbow
a- Medial collateral ligament (true)
b- lateral collateral ligament
c- annular ligament
d- posterior cruciate
13-X-ray elbow with anterior and posterior fat pad with no fracture line;
history of trauma:
a- fracture head radius+ effusion
b- supra condylar fracture
c- elbow effusion
d- elbow sprain
14-Fracture around ankle
a- Stress position for evaluating spiral fracture of tibia
b- AP more valuable than lateral in evaluation of talus osteochondral
fracture
c- middle cuneiform more seen alone
d- axial view to axial stress calcaneal fracture
15-MCQ the most common complication of bicondylar fracture of the
mandible with fracture of the symphsis menti
a- Dysphagia
b- loss of teeth due to loss of blood supply
c- non union of the symphsis menti
16-About Lee Fort injury image (le fort I)
-4-

17Hyperextension cervical injury


a- Stable
b- fracture spinous process of dislocated vertebrae
c- Diffuse thickening prevertebral space
d- sensory affection lower limb more than upper
18-Which cause sublaxation in cervical intrafacetal joints
a- chance fracture
b- clay shuover,s fracture
c- hyperflexion sprain
d- hang man fracture
19-MCQ: cervical spine injury:
a- 60% will be multiple
b- ICA injury at C1 C2
c- odontoid process fracture the most common
d- most of the fracture at C1 C2 level
Mostly / predominantly occurs at 2 levels. the most common site of injury was
the atlantoaxial region, with the most commonly injured levels in the subaxial
cervical spine being C6 & C7.
http://www.ncbi.nlm.nih.gov/pmc/articles/PMC2989526/
20-Skeletal syurvey searching for fracture especially thoracic vertebrae
a- Radiographs to demonstrate ocult sternal fracture (my choice)
b- Strong association of sternal fractures with D4-L4 spinal injuries
21-Regarding post operative spine how DD between disc reminants and
scar
a- Posteriolateral indentation through thecah sac
b- Retraction of thecal sac
c- ill defined round border mass
d- mass effect
Scar mostly involves larger area, show homogenous enhancement...while
disc may take ring / peripheral, non homogenous or no enhanncement... the
retraction of the thecal sac favous epidural scar, although ehancement is the
most reliable criteria. Other features that can help are signal intensity i.e disc
follows the signal of parent disc and is usually located ventrally.

-5-

On unenhanced MRI: epidural scar is isointense relative to the disk on T1W


& hypointense on T2W. Epidural scar unlike disk herniation does not displace
but attracts the epidural fat, nerve root, epidural veins, and thecal sac.
22-Female 70 ys RCC follow up found: calcification of ant. spinal ligmanet
spares disc spaces & SIJ:
a- DISH
b- Metastasis
c- AS
23-MCQ (X-ray of lower radius large lytic
lesion
a- ABC
b- GCT: lytic, bubbly & expansile
c- Osteosarcoma
I think ABC, some septations were apparent within.

-6-

N.B If the patient image after closure of


epiphysis it will be GCT ... if young age ABC & OS
could be but with aggressive pattern of OS we can
differentiate both of them.
Another differentiating point of GCT from ABC is
the subarticular location of GCT, you will see a
thin cortex or no cortex on subarticular side.
While in ABC you can see the bone on subarticular
side.
24-X-ray in 70ys man = Chondrosarcoma distal
femur (old pt)
chondosarcoma was expansile with cortical
interruption and chondroid matrix
25- ChondroSarcoma:
a- Hypervascular
b- good Prognosis if have ring of enhancement on CT and MRI
c- Osteochondroma more suggestive of malignant transformation if Cap
thickness > 20 mm
26-Spiral fracture.13 y old.Other
a- osteosarcoma
b- metastasis
c- Ewings sarcoma distal humerus
Ewings was involving metadiaphysis lytic with periosteal
reaction
27-Osteosarcoma:
a- Is the comonnest bone tumor
b- occur most commonly before 30 years and after 10
years

-7-

c- Commonly associated with systemic symptoms and pathological fractures


28-In para-osteal osteosarcoma
a- cause permeative cortical destruction
b- originate intramedullary
c- pain is character
d- predilection for pelvic bone
29-Radiation induced bone malignancy.
a- Osteosarcoma
b- malignant fibrous histcytoma
c- fibrosarcoma
30-MCQ, bone ABC, site of it,,,,,
They are typically eccentrically located in the metaphysis of long bones,
adjacent to an unfused growth plate.
Although they have been described in most bones, the most common locations
are:
long bones: 50-60%: typically, of the metaphysis
lower limb: 40%
tibia and fibula: 24%, especially proximal tibia
femur: 13%, especially proximally
upper limb: 20%
spine: 20-30% especially posterior elements, with extension into vertebral
body in 40% of cases
8 % sacrum

31-

X-

31
ray tibia in male
patient
complaining of irregularity at the anterior aspect of tibia with no
history of pain
-8-

a- adamantinoma
b- non ossifying fibroma
c- metastasis
32- X-ray, Expansile lytic lesion shaft of metacarpal bone
a- Hyperparathyroid
b- metastases
c- Thyroid
d- other
33-Cancer prostate metastasis
a- sclerotic
b- osteolytic
33- Cancer prostate metastasize to bone
a- erosion of anterior vertebrak body
b- calcaneus metastasis
c- sclerotic metastases
d- to skull vault
34-Most common mets in 70y old female
in metacarpal bone osteolytic from
a- breast
b- colon
c- lung ca.
Mets distal to elbow and knees...most
common lung and breast 20%. As mcq is
about female, so I think it is right choice
in female breast in male lung. However, being lytic is more towards lung ca. not
breast. (get through MCQ)
35-Regarding metastatic bone tumors:
a- X-ray bone survey is done in all patients with known malignancy. (F)
b- in 1-5% of patients, primary maliganncy could not be identified. (t)
c- Bone metastasis of ewing sarcoma has similar appearance to metastases
from leukemia on plain X-ray.
36-Typical features of multiple myeloma include:
a- A 5-year survival rate in excess of 50%.
b- Ten to 20% of cases demonstrate BenceJones proteinuria.
c- Amyloidosis is reported in about 20% of patients.
d- Complete absence of lesions on scintigraphic imaging. A periosteal
reaction.
The correct answers:
- 5-year survival 20%
-9-

- Bence Jones proteinuria (50%)


- Secondary amyloidosis in 615%
- 10% of lesions only detected on bone scans
37-Image of a woman has got bck pain; what is the diagnosis:
I wrote spinal metastases and bony metastases
Female patient with history of breast cancer MRI of the spine shows
canal and vertebrae lesions (Metastasis)
38-Bilateral renal enlargement + distal radial lucent sub-epiphyseal line +
exrtamedullary haemopiasis:
a- Leukaimia
b- Lymphoma
c- AR PCKID infantile
d- renal osteodystrophy
Leukemia is one of myeloproliferative
disorders
which
cause
extramedullary
haematopoeisis (one of sites of EMH is
kidney)
39-Case of long bone may be femur with soap bubble appearance:
a- metastases
b- fibrous dysplasia (ground glass appearence)
c- bone cyst (if ABC ==is correct----if UBC/SBS----== no, then the
answer would be a. (metastatic RCC)
Soup bubble appearence if it is Expansile and lytic (soap-bubbly) is
charecteristic for metastatic Renal cell, ABC or GCT
40-Image of MRI knee:
Pigmented villonodular synovitis
41-Regarding pigmented villo nodular synovitis
a- affect femeal 5 times more than male
b- X the periarticular pain with terminal
aspect osteoprosis is the earliest sign (Bone
density and joint space are preserved until
late stages) ----
c- elbow is the commonest site
On X-ray, features are relatively nonspecific with
appearances mainly being those of a joint effusion. Bone density and joint
space are preserved until late stages. No calcification seen. Marginal erosions
may be present but it is not possible to distinguish PVNS from synovial
chondromatosis (non-ossified synovial osteochondromatosis) Radiopaedia

-10-

42-image of mri knee -Synovuial osteochondromatosis


43-

MRI image knee


a- osteochondritis dissicans
b- ACL tear
c- septic arthritis
44-Image knee joint
a- ACL injury
b- osteomyelitis

http://www.med.nyu.edu/.../Rads%20clinics_MRI%20knee.. &
http://www.radiologyassistant.nl/.../knee-non-meniscal...
45-Shoulder MRI ant lab tear, T1 with intra articular contrast, and cause
dislocation
N.B Bankart lesion i.e. anterior labral tear with anterior dislocation
http://w-radiology.com/mr-arthrography-shoulder.php

-11-

46-X-ray pelvis AP & frog for adolescent female with hip pain what cauase
her abnormality: == on sight evaluation according to image.
a- family history
b- obesity
c- sickle cell
d- Corticosteroid use if increased
density of femoral heads, air lucency or
collapse.... etc signs of AVN
I think the question was an x ray image of pelvis
showing left hip AVN and asking about the
predisposing factors, and I think we should look
at the X-ray carefully if we found osteoporosis so the answer will be steroids, but if
we found H SHAPED VERTEBRA the answer will be sickle cell
X-ray pelvis adult female right hip AVN predisposing factor (ask about
corticosteroid not family history
47-Regarding osteoarthritis
a- PIJ is most affected in hand
b- knee most common
c- less osteophytosis than rheumatoid
d- 1ry osteoarthritis of shoulder common more than secondary with no risk
factor * (not sure)
knee is the commenest site
48-Regarding osteoarthritis
a- Involving DIP is common == Haberden's nodes
b-Knee joint is the commonest place
c- another option about differentating rheumatoid from degenerative
49-(X-ray hand) middle aged female with chronic hand pain
sure rheumatoid artheritis
50-Regarding rheumatoid
a- atlanto-axial sublaxation caused by involvement of transverse
ligament of C1 .
b- others
51-Most common cause of erosion lateral
of clavicle
a- RA
b- ankylosing spondylitis
c- langerhans histocytosis
d- hypoparathyroidism

-12-

52-HAND female 60 ys psoriatic:


a- RA: leads to displacement, disfigurment, dislocation & deviation then
and more than destruction.
b- Erosive osteoartheritis. Generally, psoraitic arthritis is one of the
erosive arthritis.
c- Gout
Erosive O.A
53-X-ray hand the 2nd MCP joint of ?right hand showing erosion with soft
tissue swelling.
a- RA.
b- OA.
c- Gouty arthritis
d- CPPD
54-For diagnosis of osteoporosis:
a- T score < 1
b- T score < -2.5 I choose this
c- Z score < 1
N.B T scor of normal (0 to -1) osteopenia (-1 to - 2.5)
osteoporosis (> - 2.5 )
Gouty
55-Image of a child does not eat vegetable what isArthritis
the
the diagnosis:
scurvy

-13-

56-Pathognomonic for hyperparathyroidism


a- hair on end
appearance
b- subprriosteal
resorption

57-X-ray skull (hair on


end appearance
aHyperparathyroidism
b- thalassemia / sickle
cell anemia (The
answer)

58-Large hand with large soft tissue


Acromegally decide according to image
59-MCQ about bad heel thickness
N.B. normally < 21mm. Increased in:
acromegaly
myxoedema
callus formation
phenytoin therapy
obesity
peripheral oedema
A useful acronym for remembering the causes is MAD COP.
http://radiopaedia.org/articles/heel-pad-sign
60-Mcq fibrous dysplasia associated with
a- Rickets and gout osteomalacia
b- rheumatoid artheritis
c- monostatic type with takyasso syndrome
d- monostatic type with McCune-Albright syndrome
e- cardiomegally
N.B. Although fibrous dysplasia is usually sporadic; a number of associations
are well recognised:

-14-

McCune-Albright syndrome in 2-3% of cases with the polyostotic form


isolated endocrinopathy without the full McCune-Albright syndrome
precocious puberty in girls.
hyperthyroidism
hyperparathyroidism: renal stones, calcinosis
acromegaly
diabetes mellitus
Cushing syndrome: osteoporosis, acne
growth retardation
Mazabraud syndrome - soft-tissue myxomas (rare); typically, multiple
Intramuscular lesions in vicinity of most severely affected bone
(http://www.jci.org/articles/view/18399)
Hypophosphatemic rickets/osteomalacia is recognized as a rare complication
of FD/MAS so the correct answer is rickets ????
Hyperthyroidism, acromegally, Cnshing's syndrome, gynaecomastia &
parathyroid enlargement have all been reported in association with polyostotic
fibrous dysplasia. (Sutton)
61-X-ray show knee joint says toddler presented with bowing of legs
a- Leukemia
b- Syphilis
c- rickets
d- blount disease
http://radiopaedia.org/articles/blount-disease &
http://radiopaedia.org/articles/rickets &
http://radiopaedia.org/articles/leg-bowing-in-children
N.B Bowing of legs in toddler, commonest cause blounts
disease
N.B Blount disease (also known as tibia vara) refers to a
local disturbance of growth of the medial proximal tibial
epiphysis. The condition is commonly bilateral. .... so if the
lesion affecting both articular surfaces we can exclude blount as it affects
medial of proximal end of tibia
lateral tibia cortex medial proximal tibial physis blount
bowing tibia rickets strait
N.B rickets (fraying, cupping of metaphysis)

-15-

N.B Commonest cause for bowing in a chlid is phsiological bowing, which


corrects at 3 yrs, bowing due to blounts disease will progress
N.B rickets =cupping etc blount=tibial metaphyseal changes developmental
blowing appears normal except bowing
N.B leukemia

62-About hip joint pathology in


childeren
a- Perth disease age between 6-9 months
b- Sliped capital epiphysis common between 6-8 years
63-X-ray knee in 12 years old patient with mild trauma severe pain &
swelling of knee joint hotness with previos history of the same
complaint with previous mild trauma long history for child with knee
pain & knee X-ray show malalignment
of the knee joint with irregular surface
and enlarges femoral condyles and wide
notch what is the cause
a- Hemophilia = flask shapped
distal femur
b- thalathemia
c- sickle cell

-16-

d- rickets)
according to image hemophilia.
64-3 Images: X-ray DLS of a child AP & Lat and pelvis. I could not notice
any abnormality except defect of posterior neural arch of S1 & relative
increased bone density.
a- osteogenesis imperfecta.
b- thiopental dysplasia.
c- osteopetrosis.
d- spondylo epiphyseal dysplasia congenita.
Osteopetrosis
Type I: Sclerosis of the skull mainly affects the vault with marked thickening;
the spine does not show much sclerosis.
Type II: Sclerosis is found mainly in the base of the skull; the spine always has
the rugger-jersey appearance,
Spondyloepiphyseal Dysplasia
posterior wedging of vertebral bodies giving rise to oval, trapezoid, or pearshaped vertebrae, as seen in the image below. The ossification of the bodies
may be incompletely fused, as depicted in frontal projection. In adolescents and
young adults, end plate irregularities and narrowed intervertebral disk spaces
become obvious with an increased anteroposterior diameter of the vertebral
bodies. Lumbar lordosis is usually exaggerated. Progressive kyphoscoliosis
may develop in late childhood. The most marked abnormality is usually at the
thoracolumbar junction, where gross ventral hypoplasia may be present.
65-MRI spine Sagital and axial:
of congenital anomalies of
the spine
a- meningiocele
b- meningiolyomelocele
c- lipomenengiomylocele

d- dural lipoma with external


sinus
66-Mcq the last site of ossification center at elbow joint appear
a- medial epichondyle
b- lateral epichondyle
c- troclea
d- capitulum
67-Enlarged vertbra in
a- potts
b- fibrous dysplasia
-17-

c- gaucher
Enlarged vertebra: Common: Acromegaly, Paget's disease, Uncommon Benign bone
tumor (eg, hemangioma, aneurysmal bone cyst, giant cell tumor), Compensatory
enlargement from non-weight-bearing (eg, paralysis), Congenital enlargement,
Fibrous dysplasia, Hyperphosphatasia.
68-Anterior scalloping of vertebra (causes)
a- Retroperitonial LN (leukemia, lymphoma, TB)
b- aortic aneurysm
c- Down syndrome
69-Delayed skeletal maturation (causes)
a- Hypopituitarism
b- hypothyroidism
c- hypogonadism
d- DM
e- rickets.
Skeletal maturation is controlled by: thyroxine, adrenal steroids, gonadal
steroids excess of these acclerates maturation & defaciency causes delay.
70-Causes of delayed skeletal maturation
Inflammatory bowel disease---------in another sentence Chron's disease
71-(MCQ) Bullet shaped vertebrae seen in:
a- Morquios disease. (true)
72-MRI shoulder, 3 axial, 3 coronal ... ask about the lesion
a- supraspinatus tear
b- labral tear
c- acromion impingement
d- fructure humerus. The fracture is not seen but bone marrow edema
is noticed
73-Blunt trauma to the neck
a- Dissection of external carotid is commonly asymptomatic
b- Dissection of the vertebral artery occurs at the level of C1-2. (Sure)
c- Multiple cervical spine levels injury occurs at same time
d- Fracture odontoid process commonly at tip
74-The 1st case Lt hip & knee pain with coronal MRI T1, T2, STIR I
saw bright signal in STIR in the head of the left femur and T1 seem
normal choices are:
a- Avascular necrosis
b- stress fracture

-18-

75-X-ray for a child's wrist with history of torsion pull of the hand and
limitation of movement, the image shows small buckle fracture at distal
radial metaphysis away from and not reaching the epiphyseal plate and
the choises were:
a- Salter harris type I
b- type II
c- Torus fracture (this is another name of buckle
fracture)
d- ? I don't remember

-19-

B) GIT
1- What separate medial & lateral segment of Lt lobe:
a- ligamentum teres
b- left hepatic vein
c- falciform ligament.
2- What separate medial & lateral segment of lt lobe (ligamentum teres not
in choices)
a- right hepatic vein
b- left hepatic vein
c- falciform ligament
3- Rt Gastroepiploic artery is a branch of
a- SMA
b- Celiac
c- IMA
d- Gastrodeuodenal a.
4- MCQ Ligament that passes throgh foramen between greater sac &
lesser sac
a- Ligament of Treitz
b- hepatoduodenal ligament
5- Which structure is retroperitonum
a- Ceceum
b- Appendix
c- Sigmoid
d- ascending colon.
The appendix, transverse colon, and sigmoid colon have a mesentery (called
mesoappendix, transverse mesocolon and sigmoid mesocolon, respectively), but
the ascending colon and descending colon and the rectum and anal canal are
retroperitoneal; the cecum does not have its own mesentery and is mobile,
owing to attachment to the mesentery of the small intestine.
6- Liver cirrhosis (MCQ)
a- hypoecoic in US
b- caudate lobe hypertrophy
c- relative Rt lobe enlargement
d- hypoattenuating in CT
7- Regarding cirrhotic liver
a- enlarged left lobe common
b- dilated portal vein
c- esophageal varices common

-20-

d- portal vein thrombosis common


8- MCQ: CT of hepatic hypoechoic lesion with
marginal enhancement (abscess).
This is CT image. However, hypoechoic liver lesion with
enhancing wall includes hydatid cyst, simple cyst in
liver, abscess.... etc according to the question.
9- A case of hepatic focal lesion large enhancing in
arterial phase and iso dense in later phases on a healthy liver:
a- HCC good arterial enhancement with rapid washout in delayed phases
(holes)
b- hemangioma best seen in portal and delayed phases, contineous filling in
(closing iris sign)
c- Renal metastases (mostly the answer)
FNH (focal nodular hyperplasis)
Nephrographic (delayed 80-180 sec) phase CT image fails to show these
lesions. Since hypervascular lesions cannot be visualized during the
nephrographic phase but can be easily seen during the arterial phase.
Nearly all patients with liver metastases from RCC have metastases in other
locations (e.g. pancreas, L.N., lung...etc).
10-Triphasic CT abdomen in adult female; lesion enhancing only at the
arterial phase
a- HCC
b- hemangioma
c- intrahepatic cholangiocarcinoma.
d- Metastases from RCC
Options to be selected: FNH, adenoma, THADs
(transient hepatic attenuation differences),
HCC, hypervascular mets (from RCC,
carcinoid, melanoma, thyroid, breast.... etc)
11- MRI hepatic lesion in a female the lesion
has a large central scar, liver not cirrhotic,
female, no bile duct diltation, central scar low signal in T2, some high
signal seen inside lesion in T1
a- HCC
b-FNH
c- cholangiocarcinoma
d- cavernous haemangioma

-21-

12-MRI liver triphasic in a female, old age > 60y showing heterogeneous
enhancement in arterial with delayed wash out, but I see dilated biliary
radicles
a- HCC would be correct if compressing porta hepatis biliary outflow,
especially in presence of liver cirrhosis.
b- Cholangiocarcinoma would be an option but its delayed phases is at 5
minutes and it shows peristent contrast enhancement in portal and 180
sec delayed phase.
c- FNH
d- cavernous hemangioma
13-CT of enlarged liver & heavy hepatic fatty infiltration with sparring
area
a- heavy infiltration
b- diffuse malignancy
c- Budd Chiari
http://radiopaedia.org/articles/budd-chiarisyndrome-1
14-(MCQ) Regarding buddchiari syndrome
on sonography is:
a- Dilated portal vein.
b- Lt. lobe hypertrophy.
d- Intrahepatic collaterals. (true)
c- peri-oesophageal varicose veins
15-(MCQ) triphasic study of the liver, asking about the timing of protocol:
a- 20-25, 50-55 and 3-5 min.
b- arterial (20-30 seconds), PV phase (60-70 seconds), delayed (3-5
minutes) (true)
16-Liver masses
a- hepatoblastoma is the most common malignant in infant
b- others
17-MCQ: about liver tumors:
a- Hebatoblastoma is the most common type in neonate
Hepatoblastoma is the most common primary hepatic malignancy in childhood,
accounting for 43% of all pediatric liver tumors
http://emedicine.medscape.com/article/940516-overview...
Commonest benign is: hemangioendothelioma, commonest malignant <2y is:
hepatoblastoma, commonest malignant >2y is HCC.

-22-

Ped. benign liver tumours are relatively rare. The list in descending order of
frequency is:
infantile haemangioendothelioma
mesenchymal hamartoma of the liver
FNH
hepatic adenoma
nodular regenerative hyperplasia
18-Liver tumors
a- HCC accompanied with cirrhosis more than
b- FNH have low T1 and low T2 with central scar
c- fibrolamellar HCC occur in old age
d- others
19-According to hepatic neoplasms:
a- HCC more with hepatitis C than B
b- FNH are common in elderly women
c- cirrhosis common with fibronodular HCC
d- FNH commonly associated with female taking oral contraceptive pills

20-MCQ: hepatic masses can be differentiated (benign/ malignant) by:
a- US
b- 1ry or 2ry by US
c- US finding correlated with histopathology. I see this.
d- focal hyperechoic lesion may be metastasis (true)
21-A cavernous haemangioma typically:
a- Of low attenuation relative to the normal liver on NCCT.
b- Has a poorly defined edge NCCT.
c- Shows rapid complete enhancement on CECT.
d- Is hypoechoic in U/S.
e- Shows increased uptake on a colloid isotope scan.
22-According to giant heamagioma
a- takes peripheral nodular enhancemet and not complete to central

23-MRCP, CT liver child with fever and jaundice:


Caroli disease.

-23-

24-Regarding to gall bladder


a- Porsalin GB predispose to carcinima
b- Emphysematouse GB is exclusivly seen in DM
c- fluid around the wall is surest to perforation
d- abscent GB stones denoting no evidence of cholycystitis in sickle cell
anemia
25- Accessory spleen
a- may occur in thorax
b- wandering spleen commonly seen in female
c- cysts mostly infective
d- occur in 60% of population
e- in 0.5-1% of people spleen located retrorenal
26-Ectopic pancreatic tissue of stomach seen in:
a- Greater curve
b- lesser curve
c- anterior wall of antrum
d- fundus
An ectopic pancreas appears as an extramucosal, smooth, broad-based lesion
either along the greater curvature of gastric antrum or in the proximal
duodenum.
http://radiopaedia.org/articles/ectopic-pancreatic-tissue
27-CT: septated cystic lesion of pancreatic body & tail
a- Mucinous cystadenoma
b- Pseudocyst
c- islet cell tumor
28-In pancreatic cystic mass
a- represent 40-60% of pancreatic tumors
b- mucinous cystadenoma is usually in head of
pancrease
c- in Pseudocyst there is enhncement of
intracystic septa.

-24-

d- intra cystic papillary projection may be high intense in MRI T1

29-Pancreatic tumors:
a- in the head + neck + uncinate process
b- most of it is associated with bile ducts hydrations
c- tumors mor than 5 cm need surgery
30-Regarding to pancreatic tumors:
a- 90% with obstructive jaundice
b- tumor in head
c- Dynamic CT better in diagnosis of vasculartity
31-Plane X-ray for esophageal atresia.
32-MCQ: Regarding to neck and esophagus imaging
a- LLO images for motility disorder
b- Cricopharyngeal indentation is at the level of C5/6).

33-Anterior indentation of the osophagus
a- pulmonary sling
b- double aortic arch
c- pulmonary aneurysm
Causes of posterior indentation of esophagus: 1-aberrant
SCA, 2-double aortic arch, so I think the answer will be
pulmonary sling
34-MCQ: about Boerhaave syndrome vs. Mallory Weiss tear patient with
chest pain, fever, pneumoperitoneum, no haematemesis
a- booerhaave
b- mallory weis
c- intramulral eosphageal rupture
d- eosphageal carcinoma
The difference is that: in mallory-weiss the tear is mucosal and pt presented
with upper GI bleeding. But in Boerhaave's esophagus is perforated through
all layers, which basically leads to mediastinitis, which is potentially lethal &
manifests with very severe substernal pain initially and then fever, leucocytosis
etc...
35-MCQ regarding to CHPS
a- it presents at 2 to 8 weeks of life
b- it presents with intestinal obstruction in X-ray
c- X-ray with contrast imaging plays minor rule in its diagnosis
d- US cannot differentiate stenosis from pyloric spasm.

-25-

36-X-ray: Dudenal atresia double-bubble sign

37- CT: gasteric exophytic mass


a- adenocarcinoma most common gastric 1ry malignant tumor
b- lymphoma --tends to be circumferential
c- pancreatic pseudocyst
d- not remember 4th choice, GIST was NOT in choices.

Schirrhous type

Leiomyoma

Leiomyomsarcoma

38-Neonatal abdominal X-ray in ICU shows dilated small bowel loops:


a- Necrotizing enterocolitis (true)
b- Meconium peritonitis.
c- Hirchsprung.
d- Constipation
39-Regarding to necrotizing enterocolitis:
a- Common in the proximal intestinal bowel.
(terminal ileum, cecum, Rt colon)
b- Increased wall thickness of the bowel & thumb
printing appearance of the wall are diagnostic.
(thumprinting is not pathognomonic sign & it
refers mostly to ischemic colitis / ishemic bowel
disease)
c- Common in a full term baby at 24 hours after birth (premature 2-3 days
after labour).
d- d- Portal vein gas is needed emergency surgical interference.
(immediate if pneumoperitoneum) Ominous sign
40-MCQ for barium follow through most likely for Crohn's.
-26-

41-Female 25y diarreah + abdominal pain


a- Crohn's
b- lymphoma
c- ischemic coilitis
42-A case of middle aged male with weight loss & changed bowel habits.
images of barium (I can't identify anything), axial cuts with regularly
thickened bowel loop mostly illeal loops:
a- TB
b- Crohn's
c- lymphoma
If colon ca. in options, it would be true.
43-Axial CT bowel lymphoma
44-Suggesting Crohn's more than UC
a- Rectal affection
b- perianal pigmentation
c- small bowl skip lesions
45-MCQ regarding to Crohn's disease: it is suspected when the patient has
a- perianal fistula and sacroiliac joint arthritis
46-CT abdomen axial & coronal showing dilated bowels with multiple
masses in abdomen with history of child 2 years making liver transplant
with biliary atresia, now complaining of vomiting, abdominal pain the
patient diagnosed as intestinal obstruction which due to:
a- metastasis
b- lymphoma (no LN is present)
c- Crohns disease
d- post transplant lymphoid hyperplasia. I think it is the true
46...intussuception...9 small CT images with dilated bowel loops and loop
inside loop sign in the lt iliac region
47-Regarding to small bowel:

-27-

a- Mets to SB is a common cause of obstruction (false: rare).


b- Calcifications in gastrointestinal stromal tumors is very common
reaching up to 80% (false: Calcification is uncommon and occurs in
about 3%)
c- Ascariasis ranges from 15:35 cm (true).
d- Polyps in CronkhiteCanada are neoplastic (false: about 15% of CCS
patients develop malignancies as CCS may be a premalignant condition
for gastric cancer, as well as for colorectal cancer).
48-CT: Intussesption Rt illic fossa
a- ilieo illial intussesption.
b- illieo-colic commonest
c- colo-colic
49-CT: Intussesption Rt illic fossa
a- ilieo illial intussesption.
b- tortion ovary
c- Ectopic kidney
Intussusception can occur essentially
anywhere, although in children most
common is ileocolic (75-95%), presumably due to the abundance of lymphoid
tissue related to the terminal ileum & the anatomy of the ileocaecal region.
Ileoileocolic is the 2nd most common.
ileoileal & colocolic are uncommon
50-2y boy, 2 months after liver transplant due to biliary atresia, presented
with IO multiple axial & coronal CT
intestinal obstruction
a- Intussusception
51-Old man US inguinal mass with iso-ecchoic well defined mass takes
color by doppler with monophasic flow by doppler choices are:
a- incarcerated inguinal hernia
b- recent coronary interventional study= hemtoma from
snapping area
c- metastatic lymph node of distally chondrosarcoma
d- spontaneous hemorrhage
Groin mass in female patient
Lymph node
Hematoma from previous coronary angiography
hematoma
52-CT abd & pelvis IO in female with hernia MCQ type of hernia:
a- umbilical
b- inguinal
-28-

c- obturator -----
d- femoral
At the level of symphsis pubis to diff. inguinal from femoral hernia (mostly with
femoral hernia; hernial sac is lateral to the pubic tubricle (arrowhead),
elliptical shaped & comprssing the femoral vein whic only considered when its
diameter is <2/3 of the diameter of the contralateral vein.)
These
hernias
occur
just
below
the

inguinal ligament, through a naturally occurring weakness called the femoral


canal. Femoral hernias are more common in women (4:1), usually elderly and
frail, although they can happen at any age, including children. Cough impulse
is often absent. The lump is more globular than the pear-shaped lump of the
inguinal hernia (which is far more common in men, 25:1). The bulk of a
femoral hernia lies below an imaginary line drawn between the anterior
superior iliac spine and the pubic tubercle (which essentially represents the
inguinal ligament), whereas an inguinal hernia starts above this line. It occurs
medial and adjacent to the femoral vein and usually contains peritoneal and
greater omental fat, and some-times bowel loops (in this case, only fatty tissue
is present in the hernia, no bowel).
The potential for incarceration and strangulation in femoral hernias is
higher than for all other hernias combined! Thus, a high index of suspicion
needs to be maintained in acute onset of a tender groin mass. CT scan is the
imaging modality of choice.

-29-

53-Colonic diverticulosis what is common:


a- short colon
b- diverticulae are more prominant in enema durig diverticulitis.
c- the rectm frequently involved (rarely)
d- proximal colon 1% involvement (40 %)
Diverticulosis can present with painless rectal bleeding as bright red blood per
rectum. Cramps and tenderness may also occur in the affected areas, comp:
infection,bleeding, perforation more in sigmoid
Both single & double contrast barium enemas: are able to demonstrate
diverticula as barium-filled out-pouchings. When seen en-face they can look
similar to polyps but can be distinguished by the presence of pooling contrast
within the diverticulum and forming a meniscus. Even if seen on a recumbent
overhead film, the two can usually be separated.
CT: Diverticula are usually outlined by gas. The colon may be thickened and
shortened. On CT VR colonoscopy, they are seen as complete rings delineating
the neck.

-30-

Colonic diverticula are most common in the sigmoid colon and to a lesser
extent, in the descending colon. The entire colon can be affected however, with
15% of patients having right sided diverticula In patients from Asia, Rt. sided
diverticula are more common, and can either be single or multiple.
54-According to diverticulosis: shortening of the colon
a- enlarged at barium enema in colitis examination
55-The most complicated diverticula in GIT:
a- epinehric D
b- Zenker's Divertiulum
c- 2nd part deudenal D
d- upper jeujenal D
56-Regarding to appendicitis
a- Pt with abdominal pain & appendicolith is highly suggestive of
appendicitis
b- other choices
57-X-ray abdomen supine (not erect) what is the
proplem:
a- cirrohsis
b- Intestinal obstruction
c- others
58-Intestinal obestruction
a- abscence of gas in supine film exclud obst
b- air fluid level in erect film is diagnostic
c- associated with colon dilation
d- volvlus is the common cause in adult
59-Plain X-ray abdomen (I see sclerotic L1 + pelvic calcification):
a- cancer bladder...(I think this is true) if prostatic ca. in options, it would
be correct.
b- liver cirrhosis.
c- intestinal obstruction
60-X-ray, the image is similar to this image,
but it also shows some abnormal
calcifications in the region of Rt iliac fossa,
(really I didn't see the sclerotic vertebral
bodies) which do you think the cause of this
appearance:
a- Abnormal nasogastric tube position (I
didn't see it)

-31-

b- Intestinal obstruction form adhesions (I didn't see air fluid levels, only
gas in the stomach)
c- Cirrhosis I chosed this coz I suspected ascitis and it was really like
spider appearance of the abdomen like this image
d- Urinary bladder cancer I did not suspect because I did not see sclerotic
deposits
61-Childaiti syndrome of symptoms
a- laryngeal pain in talking
b- pain increase on deep breathing
c- pain radiating to back
d- pleuritic pain
62-Childitis syndrome associated with:
a- air in the vestibule
b- air appear under diaphragm in X-ray
c- upper lobe bleb
63-Plain abdominal X-ray showing
calcification in the abdomen of a child, this
finding is due to:
a- Rahbdomyosarcoma
b- RCC of the child
c- neuroblastoma
d- Mesonephroma
64-Question about retroperitoneal fibrosis
a- May be associated with sclerosing cholangitis
b- 10 % idiopathic.
c- GIT affection is more common in patients more than 50 years old
d- duodenum is most common site to be affected.
- Retroperitoneal fibrosis can be associated with Crohn's, UC, and sclerosing
cholangitis.
- 60-70% is idiopathic
- The peak incidence in adults 40-60 years.
- Ureters are the most common to be affected.
65-Commonest cause of hemoperitoneum in neonate
a- frequent umblical cathter transfusions
b- blood disease
c- NEC
d- adrenal haemorrahge

-32-

Hemoperitoneum may be caused by some obstetrical & fetal factors; breech or


complicated delivery, hepatomegaly, macrosomy, anoxic liver congestion being
the most frequent causes. Direct or indirect trauma of an abdominal organ
during delivery represents the major pathogenetic
factor
66-MCQ what is the tumor which show >50%
calcifications on abdominal X-ray in Ped
a- nueroblastoma
67-Most common type of intra abdominal hernia:
a- Right para deudenal
b- Left para deudenal
c- Mesenteric hernia
d- Winslow's foramen
68-2nd case: Eldery patient with history of rectal
bleeding IMA angio done show like this picture
choices are:
a- rectal caner == vascular tumoral blush.
b- angiodysplasia == multiple areas of
perivascular blush & abnormal vessels.
69-Plain HASTE study of the upper abdomen
MRI show bright hepatic lesion, 2 nd image:
T2 same cut shows rim of perihepatic ascites
& occult capsular retraction and 3rd imge
shows the lesion has early arterial & delayed
enhancement + porta-hepatis LN
according to images: but this description
towards cholangiocarcinoma
a- Intrahepatic cholangiocarcinoma. (someone's answer)
b- Cavernous hemangioma
c- HCC
d- Mets

-33-

C) Genitourinary
1- Rt spermatic vein drain into
a- Rt renal
b- IVC
c- inferior mesentric.
Lt testicular vein, unlike the Rt, joins the Lt renal vein instead of the IVC.
2- Largest area of prostate
a- central
b- peripheral
c- fibromuscular
3- Pt with bilateral small hypoechoic kidneys; most common cause
a- renal vein thrombosis
b- chronic pyelonephrits
c- chronic renal impairement
d- other option.
http://radiopaedia.org/articles/renal-vein-thrombosis
Chronic renal v. thrombosis & chronic renal failure are hyperechoic , in renal
vein throbosis kidneys are enlarged in acute, small sized hyperechoic in
chronic.
4- Unilateral small kidney with multiple scar
a- DM
b- chronic ischemic changes
c- ranal vein thrombosis
Causes of unilateral small scarred kidney: 1-pyelonephritis, 2-TB, 3-lobar
infarction, 4- congenital hypoplasia.
5- Emphysmatous pylonephritis CT show air in the kidney
6- Renal US image: pateint
come with Rt. sided
abdominal pain cause:
a- chronic renal scarring
b- hypertrophied column
of bertin
c- renal angiomyolipoma
d- cortical parenchymal
defect)
I think scar. AML not a
common cause pain

-34-

accidentally discovered, not commonly but may be if Hge occur (Pt may present
with numerous other symptoms & signs e.g. palpable mass, UTI, haematuria,
renal failure, hypertension)
US Was exactly like this showing: junctional parenchymal defect DD include
AML & previous scar

7- MRI /US of Lt. PUJ obstruction & Rt. renal cortical cysts polycystic
kidney

8- Baby 6 months with bilatral enlarged kidny & distal radial lucency
a- autosomal recessive type polycystic kidney
b- renal osteodystrophy
-35-

c- leukamia
9- Less risk of develop carcinoma
a- Acquird cystic disease
b- pylonephritis
c- von hipple
10-IVU image
a- Ectopic kidney
b- agenesis
c- ptosis
11- The common cause of fatty lesion in the kidney
Angiolipoma
12-About nephroblastoma
a- Calcification more than 80 %
b- Lung mass
c- almost presented by haematuria
d- lung metastasis is common than other region
13-Regarding to duplex kidney & double ureter
a- lower ureter normal insersion
b- upper more liable to reflux
c- other two choices
Upper moety: uretrocele + increase obst + open lower medial to orifice of
lower moety
Lower moety: Vesico-uretric reflux as short course
14-Regarding to congenital anomalies of ureter
a- Lt PUJO is common
b- female megaureter has distal part dilation proximal to obstruction
c- retrocaval ureter in 20% on left side.
d- common in females
http://emedicine.medscape.com/article/450785-overview#a0199 &
http://www.uptodate.com/.../congenital-ureteropelvic...
PUJO most common in Lt side, common in males, retrocaval 0.07%,
megaureter shows distal end narrowing or normal caliber with proximal
dilatation.
15- In pregnant woman with pelvi-ureteric disease
a- dilatation of ureter is common on the left side
b- dilatation involve the whole length of the ureter
c- dilatation is relieved immediately after labour

-36-

d- persistent ureteric dilatation after delivery is not necessarily


associated with increased chance of infection (I choosed this)
16-MCQ regarding to ureteric stones:
a- PUJ obstruction is more common
b- Lower ureteric obstruction is more common
c- and 2 other options
17-IVU of asymptomatic female multiple radiolucent filling defect some
are not separable from the wall

a- stones
b- clots
c- ureteritis cystica (I choose this)
d- multiform TCC
http://radiopaedia.org/articles/ureteritis-cystica
18- Regarding Urachal tumors
a- Right side of dome
b- commonest bladder tumors
c- 80% adeno carcinoma
d- presenting with obstruction
Urachal Ca: rare tumor from urachus, it is adenocarcinoma (90%). Age 40-70
ys. SITE: close to bladder (90%). Radiological findings: midline mass
anterosuperior to vesical dome in the space of retzius
19- Transitional cell carcinoma
a- more at renal pelvis than UB
b- Incidence increased with smoking and analgesic excess
c- more in children.
20-urinary tract obestruction
a- after contrast injection extra vasation is seen distal to obestruction
b- stones less than 1cm will pass spontaneously after removal of obestruction

-37-

c- cystic dilation of column of beritini in chronic hydronephrosis will not


returne to normal size after relif of obestruction*
d- 1/3 of urinoma will show obestructionin IVU
21- MCQ, concerning urinary stone, which is right:
a- crhon's predispose to cyteain stone
b- Crhon's predispose for oxalate stone
c- uric acid stone is radiopaque, ... oxalate is answer
22- about urethera:
a- Cowper opents anto penile urether*
b- Posterior urethra diagnosed by ascending cytso
c- Vermontonum not seen at uretherography
mictu asce
ivp
23- Mcq posterior urethral valve can be diagnosed by
a- ascending urethrocystogram
b- micturating urethrocystogram
c- IVP
Voiding cystourethrogram (VCUG) is the best imaging technique for the
diagnosis of posterior urethral valves. The diagnosis is best made during the
micturition phase in a lateral or oblique views, such that the posterior urethra
can be imaged adequately
24- mcq (images of ascending
cystourethrogram)
a- preserved cloaca
b- posterior urethral valve
c- other 2 options i dont remember)
25- Image of ASCENDING
cystourthrography in child showing urinary
bladder filled with contrast and urethra
connecting to rectum by thin track (give history
of abdominal distension)
a- neurogenic bladder
b- cloacal anomaly (answer)
c- volvolus

-38-

26- urethra
a- bladder neck is typically dilated in cysto urethrogrem
b- trauma of anterior urethra is common than post
c- cysto urethrogram cannot show post urethra
d- Uniform small stricture due to non infectious etiology*
27- Regarding investigation of UT diseases
a- asending urethrocystogram posterior urethral valve
b- Renal US renal scars
c- Transvaginal US vesico-urthral fistula
d- MRI staging of pelvic malignancy
28- Vas deference agenesis : - associated With
a- left renal agenesis
b- ectopic left Kidney
c- left adrenal agenesis
There are two main populations of CAVD ( congenital absence vas defer ens );
the larger group is associated with cystic fibrosis and occurs because of a
mutation in the CFTR gene,[1][2] while the smaller group (estimated between
10 and 40%) is associated with Unilateral Renal agenesis (URA).
28- Image right ovarian cystic lesion and empty uterus and left ovarian mass
mcq
a- tubo_ovarian abscess
b- ectopic pregnancy
c- ovarian carcinoma
29- CT pelvis 3 images left adencxal mass with soft tissue density mixed with fat
and focal calcification one of the images was U/S with high echogenecity fat is clear
a- ovarian teratoma
b- other ovarian tumor
c- ectopic pregnancy
d- Teratoma/dermoid
30- Regarding ovary:
a- shouldn't exceed 5 cm in all planes post menopausal
b- increased incidence of ovarain tumors by tamoxifrn therapy
c- serous cystadenoma never occur bilaterally
31- Regarding ovaries
a- ocp increase risk of malignany
b- bilateral serous cystadenocarcinoma is rare(this is my answer)
c- ovarian size do not exceed 5 cm postmenopausal
32- regarding leomyoma of uterus
a- occur in 55% of female in reproductive age

-39-

b-Trans abdominal US detect up to 95% of leomtoma


c- MRI is more accurate in detecting size and numbers of leomyoma than
US
d- MRI T2 cannot clearly identify intramural and subserous leomyoma
33- MRI image ill defined wide transitional zone
a- adenomyosis
c- endometritis
d- normal study
e- cancer
N.B adenomyosis : The
most easily recognised feature
is thickening of the junctional
zone of the uterus to more
than 12 mm, either diffusely
or focally (normal junctional zone measures no more than 5 mm) 5.
T2
typically a region of adenomyosis appears as an ill-defined ovoid / diffuse
region of thickening often with small high T2 signal regions representing small
regions of cystic change.
the region may also have a striated appearance 5.
T1
foci of high T1 signal are often seen, indicating menstrual haemorrhage into the
ectopic endometrial tissues7
C+ (Gd) - contrast enhanced MR evaluation is usually not indicated in
adenomyosis, however if performed, it shows enhancement of the ectopic
endometrial glands.
34- MRI image of uterus (small image)
a- Endometrium hyperplasia
b- Adenomyosis
c- Normal
35- mcq) ENDOMETRIOSIS (tricky one)
a- Coarse internal echoes on ultrasound (false: low level internal echoes).
b- low signal intesnity in T1 on MRI (False high signal because of
methemoglobin).
c- May be seen in chest causing hemoptysis (true).
d- Cause infertility in 30-50% of patients. (false 20% of infertile women
undergoing laparoscopic investigation have endometriosis while 30-50% of
women with endometriosis are infertile).

-40-

e- If can erode the bowel wall in abdomen (rarely proximal to the terminal
ileum).
36- Regarding endometrium
a- Endometrial polyps are associated with tamoxifen therapy
b- endometrial thickness do not exceed 11 mm in thickness through out the
cycle
c- endometrium not seen in prepubertal females
d- endometrium is always echogenic than myometrium throughout the cycle
early prolif. 6-7 mm , late proliferative 11 mm , secretory 16 mm, post
menopausal 4-5 mm
37- Image, HSG
a- Bicrnate uterus
b- septate uterus
c- Rupture uterus
d- Fistula between uterus and colon
uterine septum cannot be differentiated from bicornuate uterus by HSG
alone.
38- HSG
a- septate
b- bicornuate and asherman
c- fistula with intestine
d- ruptured uterus
39- 2 image Female with cesarian section since 4 days and have fever and
abdominal pain choices
A-Retained placenta
B-abdominal abcess
C-pyelonephritis ---could be correct if vaginal delivery
not c.s
D-hematoma*
Really I cant see any thing except some gases inrt iliac
and others in lt hypochondrium and radiolucent area at
site of uterusI see metallic foreign body at Left iliac
fossa... so I think answer is iatrogenic abdominal abscess.
40- Ectopic pregnancy surest sign
a- cannot happen with intrauterine pregnancy
b- adnexal mass
c- thick endometrium
d- fetal pole with pulsations seen in the adnexa (100% specific but only seen in
25%)

-41-

41- Regarding placenta


a- full bladder to exclude previa(empty)
b- maximum thickness 8 cm(4 cm)
c- percreta is invading uterine myometrium
d- painless bleeding is typical for abruption(painful)
N.B regarding FRCR bladder should be empty not to be mistaken as
covering IO .... max. thickness of placenta 4 cm ...painful is typical with
abruption ... so the most correct answer is percreta although it extend behind
myometrium
http://radiopaedia.org/.../spectrum-of-abnormal-placental....
42- regarding adrenal disease
a- calcification indicates chronic infection
b- pathological fractures are common
c- other 2 choices i dont remember) ... I think chronic infection.
http://radiopaedia.org/articles/budd-chiari-syndrome-1
43- MCQ: regarding adrenal disease:
a- infection is the most cause of calcification
b- cysts is the most cause of adrenal failure
c- addison disease cause bilateral calcifications uncommon in primary Addison
disease
d- Hge in adrenal common in infant causing insuuficiency
44- adrenal
A- most common cause of bilateral ca is addison
B- pituitary dependent cushing associated with inevitable gland enlargement
C- addison due to mets in one gland
D- RT gland should appear in CT same size of RT diaphragmatic crus

45- mcq Regarding adrenal hemorrhage in neontes?


a- hypoechoic mass in us.
b- calcification 2 weeks later in xray
c- hypervascular on color duplex study
N.B Regarding adrenal hemorrhage in neonates it has hypoechoic center
on US. ..http://radiopaedia.org/articles/adrenal-haemorrhage so the answer is
calcification after 2 week as the hematoma appears inhomogenous to
hyperechoic early and hypoechoic centally later and eventually becomes
completely anechoic and cyst-like
Ultrasound imaging is preferred for neonates because the adrenals are easily
identified by an experienced observer and the babies can be evaluated without

-42-

radiation exposure or sedation. In the acute phase, the hemorrhage is echogenic and
the gland is enlarged. Interval follow-up demonstrates simple or complex hypoechoic
evolution over the next few weeks, with decreasing size and conspicuity of the
glandular
tissue.[8]
(Emedicine) http://emedicine.medscape.com/article/376445-overview#a20
Acute stage <7 days (iso-low signal in T1, low T2), subacute 7days-7 weeks (high
signal T1&T2), chronic >7weeks (low signal of hemosideren rim in T1 & T2)
46- Left adrenal lesion relative large CT image show fat attenuation with soft
tissue strands within (its density is the same like sub-cutaneous fat but with no
measurements of the House field units) next step
a- Follow up by clinician
b- MRI
c- biopsy
The presence of pure fat within an adrenal lesion at CT is diagnostic of a
myelolipoma,
and
no
further
work-up
is
required..
www.gruporessonar.com.br/k/artigos/12231999.pdf page 1009, right lower corner.
lipid-rich adenoma is not pure fat. It is less than 10, while pure fat is less than O , or
even -10 MRI in phase and out of phase to see intracellular fat...ie. Adenoma while
pure fat in CT (negative HU) is diagnostic MRI will not add, and is expensive.
7- (image) of triphasic CT of Rt. adrenal mass with precontrast HU 25, at
70 sec 97 HU & delayed 10 min showing 47 HU in a Pt. with a known
bronchogenic Ca.
a- Metastasis.
b- MRI is needed.
c- Adenoma. (true) === washout =
69.4% measured
d- Pheochromocytoma
washout can be measured as post contrast =
portal phase HU-delayed HU devided by post
contrast = portal phase HU-precontrast HU x
100= --- % washout
in this example 97-47 / 97-25 x 100 = 50 / 72 x
100 = 69.4 %
http://www.radiologyassistant.nl/.../p421ae.../adrenals.html
- N.B ** adenoma is commoner then mets even with hx of broncogenic
ca.

-43-

48- Regarding adrenal tumors


a- myelolipoma is commonest benign with malignant potential
b- pheochromocytome occur in association with medullary thyroid carcinoma ( I
choose this). = MEN
49- commonest cause of cushing syndrome
a- adrenal adenoma
b- adrenal carcinoma
c- adrenal hyperplasia*
d- neuroblatom
50- - regarding pheochromocytoma
a- 80 % malignant
b- not enhanced in contrasted CT exam
d- is high vascular tumor
d- Increase in frequency with medullary thyroid carcinoma
51 Causes of Adrenal Calcification in Adults Include
a Adrenal adenoma.
b Adrenal hyperplasia
c Wolman disease == cal. In children
d Addison's disease. = adult
e Sarcoidosis
Causes according to incidence
1-Hemorrhage
2-TB
3-Waterhouse-Friderichsen Syndrome
4-Neuroblastoma
5-Pheochromocytoma
6-Addison Disease
7-Wolman Disease
-44-

8-Adenoma
9-Carcinoma
52- Pt with transplanted kidney..time of acute rejection:
1.12 :24 hours
2-1:2 days
3-1:2 weeks

-45-

D) ULTRASOUND
25...24...20...16....15...13...9...3
1- An ultrasound hypoechoic well defined lesion in the liver the options are
a- hepatic abscess
b- Hcc
c- the other options I did not remmenber

2- Us image of gall bladder with marked posterior acoustic shadowing


a-emphysematous cholecystitis
b- stones
c- porcelain GB
image shows a linear echogenic focus (arrow)
with a reverberation artifact (arrowhead) at the
gall bladder fundus Ephsematous cholecytitis
N.B Air in the Gb lumen will cast
reverbration artefects, while caculi will cast
shadowing
N.B Marked posterior shadowing occurs mostely if GB is contracted on stones .. in
emphysematous u will see shadow+artifact (
3- Ultrasound image at the pancreas level with arrows
pointing to SMA & Splenic vein.
4- one coronal CT Image for Vascular liver anatomy bad
quality answer is a
5- image us
Gall blader stone
6- GB stone
a- 100 % of CBD stone is diagnosed by US
b- CT has upper hand
c- After ampullary dilataion surgery GB US is difficult
7- Image with 2 arrows pointing to
a- middle hepatic vein + main PV
b- middle hepatic vein + hepatic a

-46-

c- RT hepatic vein + PV
d- Rt hepatic vein + hepatic a
8- Us image of gall bladder with marked posterior acoustic shadowing
a- emphysematus cholycystitis
b- WES sign (very typical)
9- Mcq About renal 5 mm hyperechoic lesion can be diff between stone and
parenchymal sinus fat by;
a- color Doppler I shows this
b- power Doppler
c- gray scale
d- pulsed Doppler
10- image ultrasound for testicular microlithiasis

11- (mcq) U/S of testes:


a- In epidididymo-orchitis epididymis is hyper echoic , enlarged ( false ).
b- Microlithiasis is predisposing to germ cell tumor.(true)
c- epididymis is hypoechoic to testis. (False).
d- Doppler flow changes diagnostic for testicular
torsion
12- Antenatal ultrasound
a- Holoprosencephaly
b- Hydrocephalus
c- Schizencephaly
13- - Most accurate measurment for
fetal growth retardation
a- (FL/AC)
b- (BPD/HC)
c- HC/AC. increased head circumference
(HC) to abdominal circumference (AC)
ratio (in asymmetrical type)

-47-

N.B most reliable criteria to determine IUGR combination of EFW +


Amniotic fluid volume + presence of maternal hypertension+ elevated
umbilical cord systolic/diastolic ratio
14- young female complaining of pelvic pain Ultrasound image axial
and sagittal for hematocolpus

15- (mcq) regarding ultrasound:


a- Full urinary bladder wall thickness up to
3mm. (True)
b- PV diameter exceeds normally 20mm.
c- CBD measures about 10mm.
d- Gall bladder wall thickness up to 5mm
16- US image of testis showing:
a- epidemoid cyst (true)== onion
skin appearence
b- abscess
c- seminoma
17- US Image of testicular microlithiases multiple small size microlithiasis
in area of slight echogenic than the testis
a- limited microlithiases
b- classic microlithiasis
c- semmenoma
d- non-semmenoma
Classic testicular microlithiasis is defined as five or more echogenic foci per
view in either or both testes, and limited testicular microlithiasis defined as
one or more echogenic foci that do not satisfy the criteria for classic
testicular microlithiasis.
18- antenatal US showing ascitis +
thickened abdominal wall (pathognominic
for Hydropes fetalis)

-48-

19- mcq) image U/S of testes:


a- abscess haematoma
b- seminoma .(true)
20- Fibroadenoma in U/S :
a- oval (hypoechoic) mass with long axis parallel to chest wall. (True).

b- mass with long axis perpendicular to chest


wall.
c- Well defined mass with hyperechoic
nodule inside.
d- Echogenic mass with posterior acoustic
shadowing.
Regarding Fibroadenoma... I Choosed Well
Defined Capsule Around With Length
>Height
21- breast Us
a- hypoechoic lesion with posterior acoustic enhacement suspicious of
malignancy
b- hypoechoic lesion with posterior acoustic shadow suspicious of
malignancy (correct answer)
22- Fibroadenoma image
breast us -23
a- US invest of choice in implant rupture
b- US is best choice after 50 y
c- surest sign of malig loss of LN hilar fat
d- multiple intra ductal papillomatosis has malignant potential
24- Doppler,,,,, No frequency shift if perpindicular on probe face
25- Surest sign of renal artery stenosis in Doppler
ultrasound :
a- PSV more than 250.(true)
b- RI more than 1.
c- absence of diastolic waveform (it happens only when
50% stenosis).
26- Mcq Doppler artifact I shows shifting is more
common
27- image ultrasound what is the sign name it was
WES sign
-49-

The wall echo shadow sign (also known as WES sign) ) is an ultrasonographic
finding within the gallbladder fossa referring to the appearance of "wall echo
shadow," characterized by two curvilinear parallel hyperechogenic lines
separated by a thin hypoechoic space and acoustic shadowing distal to the
hyperechogenic line in the far field. It suggests either a large gallstone or
multiple small gallstones fill the lumen of a contracted or incompletely
visualized gallbladder

-50-

E) Interventional
3...a ....because was written in the choices Brodel line is posterior...not
avascular as in the file
4...b
7...b depridment
In my exam there were 4 Q in interventional and my answers was the same as
in our file but my score was 1/4
so we need to revise the answers of the following questions (numbered acc to
the file):
Q 3: why posterior approach in nephrostomy
Q 6: x-ray :hand : soft tissue edema ; the answer was depridment
Q8: X-ray chest : about tubes: ETT and chest tube
I cant remember the fourth Q as I am not sure if the question of amount of
contrast used selective renal angiography is considered in the CVS section or
the interventional section
The ideal rate of contrast injection in selective renal angiography should be at
least 50% above the mean flow to the kidney to avoid regurgitation of
contrast...so it was calculated to be about 15 ml/sec
sorry the image in the question was not for the main renal artery but selective
angiography for small accessory polar branch
1- image of percutaneous nephrostomy asking about the name of this
technique

2- image what is the procedure it was Percutaneous transhepatic


cholangiography
3- Posterior approach in nephrostomy , why?
a- colon is anterior
b- stones more common posterior
c-brodel's line is avasular
4- image ERCP, CT with pnemoperitoneum, what you will do
a- urgent surgical exploration and repair
b- conservative and follow the patient
If large tear with severe free air in peritonum so surgical.
-51-

5- patient come to emergency with acute Rt arm pain with (3 images with
catheter in axillary A. first 2 there is defect in proximal axillary artery &
good run off and third image all vessel is good visualized with mild stenosis
at the site of previous defect) what is the next to do
a- surgical repair
b- Stent
c- selected trans-catheter thrombolytic TTT
d- systemic anticoagulant
6- Image : X ray of hand with a thick sheet within the soft tissue adjacent
and parallel to the metacarpal bone showing few air loculi, what to do next
a- antibiotics
b- Debridement
c- urgent surgical consultation.
d- urgent surgical reduction
7- old male has edematouse hand with erthyma and fever
a- surgical reduction
b- surgical depridement (sure)
c- elevate the arm and compression
8- 41- image X ray chest ETT,chest tube
a- no need for chest tube
b- No need for ETT
c- ETT need to be pushed down
d- Chest tube needs adjustement
9- Bilateral MLO mammography I saw vague calcification at left upper
quadrant so ? what next action
a- biopsy
b- spot compression view
c- spot magnification view of left upper quadrant I choose this)
10- image of a hypoechoic lesion in the breast what is the next step is core
biopsy

-52-

F) Chest
1...5...15...2032...38..5354
22..... b
50... a
55...a
1- Tracheal Bronchus:
a- left upper lobe
b- right upper lobe
c- left lower lobe
d- right lower love
2- Mcq what is separating medial segment of right lower lobe from others:
a- azygos fissure
b- hemiazygos fissure
c- transverse or oblique fissure
d- Inferior accessory fissure
3- Internal mammary lymph nodes draining of chest
wall)
4- normal chest of neonate
a- thymus reach lat wall is abnormal
b- elevated left diaphragm more than rt is abnormal
C- PA is more diagnostic than Ap
D. cardiothoracic ratio in neonates is greater than
adult
5- Image AP and lateral films right upper lobe collapse
= inverted goldes S sign, elavated horizontal fissure
6- X-ray ICU with ETT tube in the right main bronchus and subsequent total
left lung collapse
my answer was total Lt. lung collapse due to malposition of ETT into Rt. main
bronchus.
7- X ray chest image of a child showing increased opacity of the left
hemithorax with ipsilateral mediastinal shift and elevation of the left
diaphragmatic copula and crowding of ribs with hyperinflation of the right
hemithorax
a- LT lung collapse due to foreign body
b- rt congenital lober emphysema
c- lt lung consolidation

-53-

8- image x ray chest AP, lat, pt young complain of not remembre diagnosis
a- lingular atelectatic band I chose
b-aortic rupture
9- Left lower cavitary lung lesion image?
Abscess with pleural effusion.
10- Regarding miliary
a- TB spare apices
b- pleural effusion
c- septal lines
d- randomly distributed nodules.
11- Indian 30 years manifestation showing Rt upper lobe
consolidation which support primary TB than 2ry TB
a- LN
b- scarring
c- cavitaion
d- others
12- According to renal TB
a- bilateral in most cases
b- 10%pulmonary active TB
c- UB calcification
d- calyceal dilatation in late stage
13- Chest Axial CT Weight Loss And Night Fever
(Small Nodules Periphrally)
A- Milliary TB
B- Tree En Bud
C- Pn
D- TB Rosen Marry
14- 1ry TB is commonly cavitaprima
b) intial lesion in 10% present in apicoposterior segment of upper lobe and
apical segment of lower lobe
c) effusion canot be the only manifestation of primary tb pleural
d) TB occur in primary more than post primary milary
15- most comon sign of 1ry TB : if consolidation is from options, it would
be correct.
a- cavity in upper lobe
b- Hilar lymphadenopathy

-54-

c-other tow not remember


16- middle aged african female with eye proplem SOB image bilateral hilar
adenopathy
A- sarcoid african = black.
B- TB
C- cancer
17- what is suggestive sarcoid than T.B
a- pleural effussion
b- calcified pulmonary lesion
c- negative mantoux test
18-xray chest patient has bilateral emphysema and complain from fever
&cough
a-carcinoma
b-metastasis
c-asparagilloma = A symptomatic (cavitary lesion in Rt)
d-emphysematousebullea
19- CT at level of alveolar margin and Pt. complain of recurrent chest
infection and fever
a- Septic emboli
b- fungal infection
c- aspergillosis
20- In lung cancer
a- 75 % is asymptomtic
b- pancoast tumor is adenocarcinoma
c- adenocarcimo is central
d- X-ray scans of lungs are apredisposing factor
21- Q .what type of lung cancer that more to cavitate
a- small cell
b- large cell
c- adeno
d- squamous cell carcinoma
22- axial CT lung base lung window+ abdominal x ray+angio show like
tumor blush at right supra renal boy 13 ys old with recurrent chest
infection theres two basal sub pleural nodules but lung window with no
definite calcification
a- immotile cilia syndrome
b- round atelectasis
c- neuroblastoma
d- hydatid
-55-

23- Regarding CT of bronchial Tree:


a- Elderly patient are common than in children with FB obstruction specially
those in ICU
b-Air bronchogram present in bronchoalveolar carcinoma
24- Regarding Lymphoma
a- extra-nodal is more common in non-hodgkin than Hodgkin
b- LNs has homogenous enhancement with contrast CT
c- calcification on LNs is more before TTT
25- mcq what is the most likely primary tumer in patient with calcefied
lung metastssis on chest x-ray:
a- Thyroid
b- lukemia
c- scc of the neck
d- not remember the 4th
26-CT chest right pleural effusion with
multiple bilateral nodules
metastasis
27- Image left lung multiple emphysematou
bullae mcq
a- congenital lobar emphysema == mostly in
LUL
b- emphysemtous malformation
c- ruptured bullae....
d- cystic adenomatoid malformation.
28-Regarding to emphysema:
a- alpha1 antitrypsin are common with panacinar emphysema

b- emphysematous bullae are centrally located


c- lower lobe bullae are less symptomatic
29- Mcq regarding cardiogenic pulmonary edema
a- cannot be differentiated from non cardiogenic pulmonary edema in x ray
b- in 75% of cases it presents with perihilar consolidation
c- it is associated with upper lobe diversion of pulmonary arteries

-56-

d- I am not able to remember the 4th choice exactly but I think it was it
represents 75 % of causes of pulmonary edema
30- Image showing tree in bud appearance & asking about tree in bud). is
it CT chest ??
31- Cystic fibrosis:
a- calcification in pancreas 30 %
b- another option about size of gall bladder
c- and 2 other options I don't remember
Cystic fibrosis (CF) is an autosomal
recessive genetic disease that affects the
exocrine function of
the lungs, liver, pancreas and small
bowel resulting in progressive disability
and multi-system failure.
A) pulmonary manifestations of CF
bronchiectasis.
pneumothorax.
recurrent bacterial infection.
pulmonary arterial hypertension.
B) ABDOMINAL MANIFESTATIONS OF CF
Distal intestinal obstruction syndrome (DIOS)
meconium ileus: 10-20%
rectal prolapse
cirrhosis and hepatic steatosis
oesophageal dysfunction / gastro-oesophagheal reflux
pancreatic insufficiency
Fatty replacement of pancreas
Distension of appendix but reduced risk of appendicitis
C) Head and neck manifestations of CF
Chronic sinusitis
Nasal polyposis
Musculoskeletal manifestations of CF
32- Regarding IPF:
a- the most common findings in HRCT is ground glass appaerance.
b- subpleural opacity at mid and upper lung lobes.
c- decreased lung volume
-57-

33- MCQ: the most accurate sign of pulmonary edema:


a- septal lines
b- consolidiation
c-enlarged heart
d- upper lobe diversion vessels
34- MCQ - Basal area with soft tissue and air Which mass in the chest show air
lucency within
a- Bochdalik hernia
b- Fat
c- pericardial cyst
d- Fat necrosis
35- Regarding Bochdalek Hernia
a- It is right sided.
b- not a known cause for respiratory distress.
C- It is a cause of radio opaque hemithorax

36- Chest X-ray in Pt with difficult swallowing


a- mediastinal mass
b- bogdalic hernia
c- zenker's diverticulum
d- other option
37- most common cause of tracheal displacement
in females:
a-thyroid enlargement.
b-thyroid inflammatory disease.
c-marked pleural effusion
38 - Cause of upper lobe lung vascularity diversion
a- Mitral stenosis or any cause of cardiogenic pulmonary
edema
b- cushing syndrome
c- pneumothorax
d- pulmonary H
39- CT image of pulmonary embolism.

-58-

40- Pulmonary embolism (can be diagnosed by V/Q mis match)


41- Regarding pulmonary embolism:
A- plain x-ray showing majority of cases are abnormal
B- CTA is limited to show sub-segmental pulmonary embolism
c- CTA is commonly showing pulmonary infaraction
d- CTA has the same accuracy as conventional angiography in the
diagnosis of sub-segmental pulmonary embolism
42- Patient with chest pain and tearing pain in the throax migrating to
throat
a- pulmonary embolism
b- pulmonary edema
c- mediastinitis
d- aortic syndrome
N.B if ther's a history of surgical interference as oesophageal laparoscopy,
mediasitinitis is the right answer. If not, the word tearing is usuallly used as a
term describing the pain of aortic dissection (syndrome)
Mediastinitis is inflammation of the mediastinum.
Symptoms include severe chest pain, dyspnea, and fever. The diagnosis is
confirmed by chest x-ray or CT. Treatment is with antibiotics (eg, clindamycin
plus ceftriaxone) and sometimes surgery.
The 2 most common causes of acute mediastinitis are Esophageal
perforation,Median sternotomy
The term Acute Aortic Syndrome (AAS) is used to describe three closely related
emergency entities of the thoracic aorta: classic Aortic Dissection (AD),
Intramural Hematoma (IMH) and Penetrating Atherosclerotic Ulcer
(PAU).Clinically these conditions are indistinguishable.CT is the most
accurate imaging modality for the initial diagnosis, differentiation and staging.
http://www.medscape.com/viewarticle/757269_3

-59-

43- CT chest
a- dessecting aortic aneurysm
b- double SVC ,
c- mediastinal hematoma
CT chest was tricky Image one cut at level
of SVC. when you look carefully to the
image you will find catheter at SVC
another catheter seen at the left side in a
smaller calibre vessel so the answer is most
probably double SVC.
44- ER patient with sever injury
presented with widening of superior mediastinum in AP chest Xray,
presented also by interscapular pain & tachycardia, the next step is:
a- -conventional or CT aortic angiography
b- perfusion- ventilation radionuclide scans for possible thromboemboli
c- Skeletalsyurvey searching for fracture especially thoracic vertebrea
d- Radiographs to demonstrate ocult sternal fracture
45- Chest xray young adult with lobar emphesyma +cardiomegally +large
pulmonary artery chronic heart disease
a- increased pulmonary venous hypertension
b- Reversed Lt to Rt shunt +PAH
c- Lt to Rt shunt + fistula
46- Image: X ray adult grossly enlarged hilar
vascular shadows with peripheral prunnning
a- Pulmonary arterial HTN
b- primary venous hypertension
c- Lung disease with secondary venous HTN
47- Image: X ray adult grossly enlarged hilar
vascular shadows with peripheral prunnning
I choosed pulmonary arterial HTN
48- MCQ: case with 1 cut CT chest showing post. segment Rt LLL large
consolidation & other axial CT neck showing thrombus at the Lt carotid
a- infective thrombus
b- pulmonary embolism
c- two other options) I think infective thrombus
Septic Jugular Thrombophlebitis & P. Embolism (jugular but not carotid)

-60-

NB: signs of thrombophlibitis includes: luminal thrombus, mural thickening


of the vessel wall & perivascular stranding of fat planes together with
vascular collaterals with or without perivascular fluid / hematoma.
49- most common sign of pulmonary thromboembolism
-hampton hump
-fleishner
-wstermark
pleural effusion
50- Image shows markedly enlarged main pulmonary trunk and both hilar
shadows
a- My answer left to rt shunt with pulmonary hypertension
b- reversal of left to right shunt with aorto pulmonary fistula
51- Case as this picture: left to right shunt with pulmonary artery
hypertension

52- image: CT abdomen chronic chest pain and shortness of breath


(I saw plural effusion+ enhancing nodules)
a- T.B
b- adinoma
c- aspergilloma
d- Mesothelioma.
53- pleural effusion:
a- transudate indicate infection (x exudates)
b- subpulmonary effusion causing lateral displacement of diaphragmatic
hump
c- blunted posterior phrenic recess indicate at least 200 cc (right 250 cc)
d- on lateral xray 20cc of fluid can be visualized.
Raised dome of diaphragm, easier to diagnose when seen on left side, but a
difference of two intercostal spaces higher on right side also raises the
suspicion of subpulmonic fluid.
Increased distance between left diaphramatic outline and the fundal gas bubble
in the stomach.
Abrupt termination of lung markings at the diaphragmatic interface
attributable to fluid being denser to the normal translucency of lung base.
The pseudodiaphragm (visceral pleural interface) appears flattened specially
at medial margins, with blunting of the cardiophrenic angle.
The pseudodiaphragm shows a more lateral peak - 'lateralisation of
diaphragmatic apex'
Crowding of lung markings at lower zones in cases of moderate to large
-61-

subpulmonic effusions.
In some cases parietal pleural calcifications may help to delineate and
diagnose the effusion.
Lateral chest radiograph may show blunting of the posterior costophrenic
recess.
54- empyema
a- air fluid level is diagnostic
b- hilar lymphadenopathy is common
c- May predispose to mesothelioma in the wall
d- Right upper lobe collapse
55- Chest x ray image showing
a- hydropneumothorax with pulmonary pathology (true)
b- pleural effusion,
56- (mcq) Elevated right hemidiaphragm in a 40 years patient:
a- Eventration is most common cause.
b- If flouroscopy shows diaphragmatic paresis, CT mediastinum should be
done.
c- Pleural effusion is excluded.
d- Ultrasound abdomen can help.
57- Lymph node calcification commonly seen in :
a- lymphoma
b- Sarcoidosis
c- sclerderma
58- Calcification in lung
a- occur in chicken pox
b- occur in giant cell pneumonia
c- in untreated nonhodgkin lymphoma
d- in sarcoid pulmonary nodule
59- fatty lesion in the chest
60- In HRCT of the lung
a- indicated in PE (chronic embolism)
b- basal atelectated artifact should done the CT with the patient prone

c- others
61- according to the HRCT chest
a- to improve the mediastinuim view the window level should be centered
below zero
b- to to improve the lung view the window level should be centered below
zero

-62-

c- to improve the mediastinuim view increase the window width


d- tomography better than high resolution in detection of periphral pul lesion
for lung window the window level is -600 to -700, Window width is 10001500.for mediastinal window the window level is 40-50.window width is 350450.
62- suddenly comatosed patient in the home with image like this
choices are
a- aortic dissection
b- aortic dissection with hemopericardium I went for the second
isolated aortic dissection

Ascending aortic
dissection with
hemopericardium

-63-

G) CNS
19...13....20....29
2....a + extracranial hge
15....co poisining
34... retinoblastoma is bilaterl in 80% because was written MM hypo in T1 and
hyper in T2
1- Vein of Troland drains into which sinus. Answer SSS
The superior anastomotic vein of Trolard connects the superior sagittal sinus
and the superficial middle cerebral vein (of Sylvius).
2- Two CT brain axial cuts diag
a- interavenrticular,subarachnoid ,parenchymal ,extraaxial Hge with
midline shift&attenuated sulci
b- intravenetricular,parenchymal ,subarachnoid hge with attenuated sulci
c- intraventricular,extraaxial parenchymal hge with mid line shift
d- extra-axial parencgymal intraventricular hge attenuated sulci.
3- CT image of the brain what are the findings
a- subarachnoide
b- intraventricular
c- intraparenchymal and extracranial (scalp) haemmorrhages with effaced
sulci and midline shift.
4- Image brain CT & MRI diffusion with ADC map low signal I shows
cerebral infarction

DWI

ADC

-64-

5- Image MRI in mutiple sequences And history of nephritic S come with


distributed conscious level.
The answer is PRES.
6- image of thrombosis of left transverse sinus with hemorrhagic
infarction.
7- Superior sagittal sinus thrombosis is associated with:
a- exophthalmus
b- subarachenoid H
c- parenchymal Hematoma
d- didn't remember 4th choice.
secondary (effects of venous infarction)
Subcortical infarctions, which may not follow arterial distribution
Corticomedullary hemorrhage is common 374
8- MCQ, male patient 54 yrs presented to the radiology center , with disturbed
conscoius , slurred speech ,hemiparesis , which modality will start :
a- CT brain without Contrast
b- MRI diffusion
c- MRI perfusion mapping
d- other choice I can't remember.
9- Contrasted CT indicated in
a- Stroke
2- Hge
c- trauma
d- Increased ICT and papillaedima
10- Image for female post-partum with
headache CT shows hyperdense area and
high signal in T1
Sheehan syndrome (pituitary apoplexy (hge

11- Regarding craniofacial trauma


a- skull fractures are seen in 20 %
of cases
b- 95% associated with epidural
hematoma.
c- a two rather than three plain films
reduce the diagnostic accuracy of fractures

-65-

d- preliminary skull radiography is mandatory prior to CT


Image mri brain preiventericular hyperintense around the occipital -12
orn
h
- Infarction
a
- Encephalomalacia
b
- Leukodystrophy
c

:cause of intracranial empyema - 13


a-sinusitis
b-trauma
c-post operative
d-septiceamia
14- according to brain abscess :
a- more common cortical
b- may be related to cyanotic heart diseases
c- others
Rt-to-Lt shunt: 12.869.4% of all cases of brain abscesses. cyanotic heart
disease is the most commonly identified risk factor for development of brain
abscess in immunocompetent patients. The incidence of brain abscess in
patients with cyanotic heart disease has been reported to range 5-18.7%
15- low signal in basal ganglia in CT imaging present in
a) tuberus sclerosis
b) lymphoma
c) HPT
d) CO posoining
16- IMAGES - CT image of clivus meningeoma (pre, post and bone window)
17- Concerning CPA lesions:
a- calcification in CT is diagnostic of acoustic neuroma
b- acoustic neuroma the only lesion eroding the IAM.
c- Bilateral lesions are with neurofibromatosis (NF II)
18- (image) MRI picture of 4thvenricle mass in relatively old age patient.
( key finding... creeping throgh cisterns)
a- Ependymoma.

-66-

b- Meningioma.
c- Brain stemGlioma

19- Common manifestation of brain metastasis


a- dura
b- arachinoid
c- parenchyma
d- ventricular
20- MCQ: neuroblastoma:
a- 90% calcification on X-ray
b- may extend through neural canal
21- image MRI (small image ) in the sella
a- Micoadenoma
b- Macroadenoma
22- image of suprasellar enhancing lesion.
23- Images - MRI images of supra sellar lipoma.
24- MRI brain 2 sagittal one coronal images the
Suprasellar lipoma
coronal image was normal but one of the saggital
images both were T1 show abnormal
bulge of the posterior midbrain
a- suprasellar lipoma
b- septo-optic dysplasia
c- brain stem glioma.
25MCQ,
proved
LT.
temproparietal region cystic lesion,
which MRI favour epidermoid than
arachnoid

-67-

a- hi T2, T*
b- High diffusion mRI
c- Low diffusion
d- high flair
N.B Restricted diffusion (High) in epidermoid but free diffusion in arachnoid as
arachnoid cyst like CSF also epidermoid is grey in PD but arachnoid is black also
Flair epidermid is high but arachnoid is low
26- Regarding colloid cyst
a- cause migraine
b- most hypointense in T1
c- Most high dense in CT
c- central calcification is a known feature
27- Mcq the most vascular lesion of brain can see signal occult
a- AVM
b- cerebral angioma
c- Cavernous angioma
d- cverno carotid fistula
N.B Cavernous malformations are angiographically occult and do not
demonstrate arteriovenous shunting
28-carotid cavernous fistula ::
a- tortuous / dilated opthalmic vein--b- enophthalmous
c- and 2 other options i dont remember
29- Image of opthalamic A. aneurysm (what is the sensation lost in this case
a- visual
b- hearing
c- taste

-68-

30- Image optic canal mass homogenous mass with hetrogenous


enhancement and pyterigoid plate erosion
a- optic nerve glioma = fusiform bulging of the optic nerve which is
indiscriminable from the mass.
b- sheath meningioma = homogenously enhancing lesion around nerve
which is discriminable from it as tram-trak appearence.
c- melanoma =intraocular, intensly enhancing lesion, high T1, T2
d- retinoblstoma = intraocculr calcified intensily enhancing lesion at retina
in a kid.
N.B the optic nerve not displaced it is involved completely
31 - Image optic canal mass homogenous mass with htrogenous
enhancement and optic nerve seen through mass(track sign )
a- optic nerve glioma
b- sheath meningioma
c- melanoma
d- retinoblastoma
32- CT brain with lobulated lesion mostly sellar reaching back to claivus
with underlying destruction of bone. the lesion shows homogenous
enhancement with no extension into orbits:
a- optic nerve glioma
b- optic nerve meningioma
c- cavernous hemangioma
33- what is this case ?
a-Optic nerve glioma
b-Optic meningioma
c-optic neuritis
d-cavernous angioma *
lesion enhancement

-69-

34-Regarding orbital tumors


b- Retinoblastoma is bilateral in 80% of cases--------.(30:35%)
c - MM hyper TI hypo T2 according to vitrus
d- optic nerve glioma in 5 deced .------------(pediastric)
e- Rhabdomyosarcoma has a peak incidence in 4th decade -----(below 15 ys))
N.B Rhabdomyosarcoma is the most common primary malignant orbital
tumor in pediatrics.(true)
35- retinoblastoma
a- ca++ is diagnostic clue
b- turbid vitrous is not common
c- early optic nerve and cranial extension
d- others
36- orbit CT image as this
a- thyroid ophthalmopathy
b- sarcoidosis
c- psudotumour
d- preseptal cellulitis

37- MCQ - regarding thyroid


opthalmopathy.
a- Involve tendinitis portion
b- MRI can differentiate between muscle involvement
c- pain is more common in pseudotumor

-70-

38- Female came with headache and


personality disorder 40 y old man came to ER
complaining of headache, nausea ,vomiting
and disturbed level of consciousness. CT and
MRI images of unilocular ,hyperdens round
lesion on the roof of 3rd ventricle typical for
colloid cyst. MRI T1 iso to hyperintense , T2
hyperintense , Deffusion hypo(free),ADCmap,
CT hyperdense , hyperdense in CT and typical
site
a- colloid
b- CSF artefact
c- choroid plexus lesion
39- 3 Sagittal images for congenital brain
showing no cercbellar herniation,post fossa not big,2 axial for level of body
of lateral ventricle
a- Corpus callosum agenesis
b- Polymicogyri
c- dandy walker (not remember its type)

-71-

H) Head and Neck


1..2...3....5...10
11 ....le fort I
1- mcq neck space end at the level of hyoid bone
a- carotid
b-retropharyngeal
c- parapharyngeal space.
2- law & Stenvers views are for
a- facial bones
b- Temporal bone
3- X ray image of the neck showing air fluid level on the right side of the
neck and CT coronal reconstruction image of the neck what is the finding
a- external and internal laryngeocele.
b- zenkers diverticulum-------midline retropharyngeal
c- thyroglossal cyst-------------anterior midline.
d- pharyngeal carcinoma----solidish
lesion + intrenral degeneration and
necrosis.
Types of laryngocoele: Internal:
confined to larynx. Mixed : has
internal and external components.
The lesion herniates through
thyrohyoid membrane into anterior
soft tissue of neck. Secondary:
internal or mixed due to
obstructing lesion e.g. tumor or postinflammatory stenosis. Pyolaryngocele:
internal or mixed with superinfection. The above case shows primary (no
obstructive lesion), mixed (intra- and extra- laryngeal) pyolaryngocoele
(infected laryngocoele). An isthmus or waist through the perforated thyrohyoid
membrane is noted (red arrows in coronal reformat). The laryngeal airway was
compressed by the lesion (green arrow in axial). The patient presented acutely
needing urgent tracheostomy followed by surgical excision.
4- Regarding para nasal sinuses
a- hellar cells is post or anterior in ethmoid
b- encephalomeningiocele is comon complicatio to operation
c- another two wrong options
Haller cells are infraorbital ethmoidal air cells lateral to the lamina
papyracea. These may arise from the anterior or posterior ethmoidal
sinuses.
5- Patient with expanding lesion of the frontal sinus high signal in T1 and
markedly low in T2
a- lipoma
b- mucocele

-72-

c- sinus bleeding I choose this


N.B MRI signal intensity is very variable and depends on the proportions of
water, mucus and protein
T1--water rich content - low signal (most common)
protein rich content - high signal
T2 --- water rich content - high signal (most common)
protein rich content - low signal
T1 C+ (Gd) - enhancement if present, only occurs at the periphery
DWI - variable
It should be noted that colonisation with fungus can lead to very low signal on
both T1 and T2 weighted sequences, mimicking a normal aerated sinus 3.
6- About infected sinusitis:
a- Commonly associated with nasal polpys
b- Air fluid level denoting infection
c- Opacity of all sinuses exclude allergy.
7- Blunt trauma to the neck
a-Dissection of external carotid is commonly asymptomatic
b- Dissection of the vertebral artery occurs at the level of C1-2 (sure).
c -Multiple cervical spine levels injury occur at same time
d -Fracture odontoid process commonly at tip
7- 3 axial CT images nasopharyngal mass with widening of
pterygopalatine fossa most obvious at the bone window the mass show avid
enhancement
a- Juvenile nasopharyngal angiofibroma
b- carcinoma
8- question about signs of fracture base of skull:
a- fluid level in maxillary sinus
b- opacified frontal sinus
c- displaced pineal body
d- aerocele
9- Most common site of blow out fracture
A- ipsilateral superior orbital rim
B- ipsilateral floor of max
C- zygomatic arch
D- wing of sphenoid
Blow out fractures ; fracture of one of the walls of orbit but the orbital rim
remains intact, inferior is most common folloed by medial, superior and
rarely lateral.
10- Skull fractures
a- stellate fractures are more easy to diagnose than linear
b- blow out fracture are associated with fracture floor of the frontal sinus

-73-

c- sutures are well defined while fractures ill defined


d- Sutures are straight and smooth
http://emedicine.medscape.com/article/343764-overview Blow out fracture is
in inferior orbital rim (roof of maxillary sinus) and med orbital wall (lamina
papyracea)
N.B regarding skull fracture, blow out fracture are in floor of orbit and in the
medial wall of orbit
11- about Le Fort injury image ( I choose le fort I )
Le Fort I fracturesalso called horizontal maxillary (lower medial&lateral
walls) fractures, separating it from the palate == Floating palat.
Le Fort II fractures also called pyramidal fractures of the maxilla (lower
lateral & upper medial walls), cross the nasal bones and the orbital rim =
floating maxilla
Le Fort III fractures also called craniofacial disjunction and transverse
facial fractures, cross the front of the maxilla (upper medial&lateral walls) and
involve the lacrimal bone, the lamina papyracea, and the orbital floor, and often
involve the ethmoid bone. are the most serious==floating face
Le Fort fractures, which account for 1020% of facial fractures, are often
associated with other serious injuries.
Le fort fractures almost always involvs pterygoid plates
12- image,, CT,, patient with bad oral hygein.....
right parotid abscess with gas densitied inside
13- Image of lateral skull x ray with multiple marginal clusters of
calcifications mcq
sturge weber disease ie: meningeal clusters calcifications

-74-

I) Cardiovascular
2....22....23...25...27...29
Q4 ...ask about PDA not ADA .....I choose a
1- Mcq right vertebral artery arising from
a- right carotid artery
b- brachiocephalic artery
c- inomminate artery
d- Rt subclivian artery
2- Batson plexus
a- lymphatics
b- arteriols
c- veins
3- Umbilical vein after labor convert to... ligamentum teres
4- MCQ , if the heart cronary circulation is Left dominant , so posterior
descending artery will be a branch from :
a- left circumflex artery
N.B The right coronary courses in the right atrioventricular groove to the
inferior surface of the heart, whereupon it turns anteriorly as the inferior
interventricular artery (right dominant).The left coronary has a short common stem
before dividing into left circumflex, which courses over the left atrioventricular
groove, and the left anterior descending artery, which passes towards the apex in
the interventricular groove. In left dominant hearts the left circumflex supplies the
inferior interventricular artery.
5- sagital CT image of superior mesentric a &left portal vein
6- mcq, about coronary arteris and their branches,, anatomy

http://www.radiologyassistant.nl/.../coronary-anatomy-and...
7- image of heart and great vessels mcq:
a- MIP
b- ct coronal reconstruction
c- digital subtraction angiography
d- MRA

-75-

Maximum intensity projection MIP


8 - Image of heart and great vessels

9- Concering aortic aneurysm the most common part affecting the aorta is
a- descending aorta
b-Lt medial wall of ascending aorta
c- aortic root beyond the coronary cusp
d- ascending aorta at aortic root.( root rt lat wall of ascending)
if its general question about aneurysm, atherosclerotic is commonest cause hence
answer a wiuld be correct. If mycotic / inflammatory / dessecting aneurysm,
answer d would be correct,
Regarding atherosclerotic aneurysm ....the frequency of involvement is distal
abdominal aorta (66%) > iliac a. > popliteal a. > common femoral a. > aortic +
descending thoracic aorta > carotid a. > ascending aorta. For mycotic and
inflammatory aneurysm Ascending Aorta.
for dissecting aneurysm stanford A ascending aorta 60% stanford B descending
aorta 40% (primer)
the most common site of dissection is the first few centimeters of the ascending
aorta
http://books.google.com.sa/books?id=o_4eoeOinNgC&pg=PA624...
image of abdominal aorta angiography asking about amount of -10
contrast and time rate of injection
120 to 150 cc rate 4 to 5 cc per second
11- mcq (ct chest image for dissecting aortic....aneurysm

-76-

12- mcq dissecting aortic....aneurysm best for diagnosis


* CT with contrast
13- Abdominal aortogram arrow point to midline vessles origin from the
abdominal aorta and going down to the pelvis I could not see the common
iliac arteries choices
a- diseased distal part of abd aorta
b- sacral artery
c- inferior mesenteric artery
14- Image abdominal aortaogram distal part of abdominal
aorta is occluded ,
a- Le riche synd
b- occluded aorto femoral stent
15- CT angi o thrombus at bifurcation of aorta
16- abdominal CT angio diagnosis
a- diseased abdominal aorta with arterial
obstruction
b- non functional renal transplant
17- Young Pt. found lost consiousness 3 CT
images
a- rupture ventricular aneurysm
b- aortic discetion with hemopericardium
c- P. empolism
Image CT with contrast -18
rupture aorta with heamopeumothorax
19 - CT chest one cut at level of SVC when u look carefully to the
image you will find catheter at SVC , descending aorta not seen
a- descending aortic aneurysm
b- double SVC
-77-

c- mediastinal hematoma
d- coaractation of aorta
20- (image) Picture of IVC venogram , the options include :
a- Double IVC(true)
b- Stenosis of distal part of IVC.
c- left renal vein occlusion.
d- Left renal agenesis.
21- SVC: syndrome.
a- commonly associated With dysphagia
b- common With lymphoma(8%) rathar
bronochogenic Ca(85%)
c- Common in small cell type of bronchogenic
carcinoma
d- inferior rib notching .
e- associated with collatral to chest wall
Symptoms that may indicate this syndrome include difficulty breathing,
coughing, and swelling of the face, neck, upper body, and arms. In rare occasions,
patients may complain of hoarseness, chest pain, difficulty swallowing, and
coughing up blood. (so 1st choice is wrong)
SC lung cancer is the most lung caner causing SVC obst. and invasion(80%)
22- Mcq.. Most common complication of myocardial infarction
b- ventricular aneurysm
b- myocardial rupture.
c- dressler syndrome
LVF( 60-70%,).Ventricular aneurysm (12-15%). Myocardial rupture (3.3%),
rupture of papillary muscle( 1%), rupture of interventrcular septum (0.52%).Dresslers 78resenti (less then 4%)
23- (45 years male) with bundle branch block with septal defect is
detected in rest after stress the defect disappear and echo and ECG is
normal and no pain
A- Artifact
b- ischemia
c- MI
what is the cardiac abnormality which common seen at 4chambers -24
:echocardiogram
a- trancua arteriosis
b- ventricular septal defect
c- TGA
the question should be like that : what is the abnormality
seen at 4chembers view in echocardiogram
VSD is seen in 4chember view .
78resent arteriosus seen in outflow tract views .
-78-

TGA seen in base view.


So, VSD is final answer
25- Mcq what that berry aneurysm site can bleed
a- anterior cerebral artery
b-anterior communicating artery
c- middle cerebral
d- posterior cerebral art
26- Giant aneurysm :
a- most of it on the base of brain
b- calcification decrease the ber cent of hemmorrag
c- stenosis in the artery before it refer to arthritis
27- Angio, aorta shows?
Fistula between IJV & RT
CCA.
28- IMA angio
a- large bowel ischemia
b- rectal angio dysplasia (sure)
c- ulcer
d- rectal cancer
29- (image) selective renal
arteriogram for accessory
renal artery to lower pole,
how we used the contrast:
a- 1-3 ml at 1 ml /sec. (true)

b- 5-10 ml at 2-3 ml / sec.
c- 5 ml / sec for total 5-8 ml
d- 5-10 ml /sec for 5-10 sec Rectal
We typically use digital subtraction
IMA
angiography (DSA) and inject aischemia
total
of 10 cc of contrast for an abdominal
aortogram (4 mL/s flow rate). This
allows the origin of the main renal
arteries to be visualized (typically
arising at the L1/L2 level), and
accessory renal arteries (if present).

-79-

CCA-IJV fistula

J) Breast
2...7....9
6 ... most suggestive of malignancy
1- Internal mammary lymph nodes draining of??
b- chest wall
2- MCQ question on 80resenting80 asking about position of,, photo
sensor,,placing as
1.near areola
2.near chest wall
3.at thickest part of breast
4.at thinnest part of breast
N.B Correct position of the AEC (Automatic Exposure Control) detector is
crucial. The AEC detector must be moveable so it may be shifted underneath
the most dense area of the breast which is usually the reteroarealoar area. With
the central region of breast placed over the center of the cassette holder, the
proper position of the detector is in the anterior one third of the breast behind
the nipple, regardless of the degree
of
parenchymal
involution.
3- image,, bilateral mamo i choosed fibro adeno lipoma
US image breast tubular echofree structure what ouwill do after--------to be reviewed
A- us follow 6 month after
B- continous normal routine screening
C- Fine Needle Aspiration.
D- true cut biobsy
5- Mammography image got retroaerolar
breast lesion showing speculated outline no
calcification which BiRADS.

-80-

4
y

BIRADS 0 >> Non informative exam (invaluable exam) as in dense breast ,bad
technique and bad compression
BIRADS I >> Negative = normal
BIRADS II >> Benign : follow up every 1year as lipoma , hamartoma , cyst ,
galactocele and fibroadenoma in patient less than 40 year
BIRADS III >> probably benign follow up every 6 months for 3 times if no
changes do as BIRADS II if changes occur do as BIRADS IV as
fibroadenoma in patient more than 40 year or macro-lobulated lesion
BIRADS IV >> suspicious malignant for biobsy speculated irregular
microlobulated lesion +or other signs
BIRADS V >> High suggestive malignant : surgery spicukated mass + othe
signs + LNs
BIRADS VI >> proven cancer
http://radiopaedia.org/.../breast-imaging-reporting-and...
N.B BI-RADS Category (5): Highly suggestive of malignancy; appropriate
action should be taken. These are lesions that have a very high probability of
being malignant and should undergo biopsy. Spiculated masses and
pleomorphic clusters of calcifications are included in this category.
6- MCQ about high risk of breast ...cancer (curve image).
a- pathognominic for malignant
b- most suggestive of malignancy
c- probably malignant
d- benign
N.B the curve is high rising followed by rapid washout.

-81-

Risk factors
increasing age
1st degree relative with breast cancer
factors increasing unopposed oestrogen load
early menarche
late menopause
nulliparity
personal history of breast cancer
personal history of a high-risk lesion on breast biopsy
atypical ductal hyperplasia
atypical lobular hyperplasia
genetic mutations
BRCA1 mutation
BRCA2 mutation
-82-

7- Recently done breast biopsy what changes happens after


one year in mammogram and reach maximum at 3 years
b- oil cyst
b- increased skin thickness
c- architexural distortion
d- other option).
N.B most common changes at initial mammographic followup were architectural distortion (48%), skin thickening
(17%), and increased focal density (parenchymal scar)
(15%). Regression analysis showed no significant change of mammographic
features in all patients during 5 years
http://www.ajronline.org/doi/abs/10.2214/ajr.167.4.8819410
N.B Fat debris from ruptured lipocytes tend to conglomerate to form a
macroscopic pool of oil surrounded by lipid-laden macrophages or foam
cells known as an oil cyst. The wall can then calcify. Most often this
occurs secondary to 83recent surgery; however this is not always necessary.
8- (mcq) About ductal carcinoma in situ:
b- Never to be palpable.
b- Commonly seen in mammography as microcalcifications.(true)
c- To Duct dilatation occur in 70 to 80% of cases.

9- Regarding breast
b- microcalcifications diagnostic for ductal carcinoma
b- speculated mass and dense lymph node is diagnostic for malignancy

c- intra ductal papilloma 83resenting by palbable mass


d- fourth option i cannot remember )
10- Regarding breast mass:
a- fibroadenoma is commonly bilateral (4 % only)
b- Breast cancer is common in the upper lateral quadrant
c- Intraductal papilloma presented as palpable mass(bloody or clear nipple
discharge. No palpable masses.)
d- High resolution US is more accurate than mammography in detection of
microcalcification. (false)
11- regarding ductography
a- duct perforation is a common complication
b- breast abcess is not a contraindication
c- can DD between bengin and malignant lesions

-83-

d- if bleeding per nipple & secreatin and mammogram is normal no need for
ductography
Most of the intraductal abnormalities on mammography is identified as
form of filling defects. These can occur from both true pathological &
artefactual (e.g. air bubble) causes. Other abnormal patterns include
fusiform or tubular dilatation of ducts: occurs with mammary duct
ectasia
abrupt ductal cut off: can occur with an obstructive distal lesion
http://radiopaedia.org/articles/breast-ductography-1
N.B Although the sonographic features of breast papillary carcinomas may
overlap with those of benign papillomas, features more specific to malignancy
include a nonparallel orientation, an echogenic halo, posterior acoustic
enhancement, and associated microcalcification. Although it is not always
possible to differentiate papillary carcinomasfrom benign papillomas on the
basis of sonography, familiarity with these features will aid in suggesting the
diagnosis
www.jultrasoundmed.org/content/27/1/75.full.pdf
12- Mammography for male patient asking about
which BIRADS category
my answer was BIRADS III & i think it was wrong it
is BIRADS IV
Regarding breast male mammography if it is
gynecomastia which is the most common complain,
it will be categorised as BIRADS II. it well appear as
retroareolar density more to be bilaterally. The
other DD is cancer which appears as exccentric mass
not related to retroareolar area with other signs of
maljgnancy and categorised as BIRADS V

-84-

-85-

K) Pediatrics
6...7...18....27....29
14...x.ray with curled NGT in cervical region
1- Image : X ray child boot shaped with cyanosis (fallot tetralogy)

(plethora + shape of heart is egg on string)

2- (2 image) of child with cyanosis


a- fallots
b- treated transposion of great arteries
c- truncus arteriosus
d- PDA
On chest radiography:
- Biventricular enlargement + right sided
truncal arch in 30 40% of cases
(extreme ceour en sabot = sitting duck
Truncus
appearance).
- Pulmonary plethora only in type 1, with
oligemia in the others (due to small size
of arteries).
3- child chest x ray oligemial normal HT
size cyanosis
A- TGA
B - Truncus arteriosus
C- tricuspid atresia
D- single ventricle
4- chest xray of cyanosed newborn with
enlarged heart
a- fallot
b-transposition of great arteries (egg on string
If isolated=Filling of AP window
Or signs of other cong.anomaly
+ plethora+ narrow mediastinum)

-86-

5- aortic coarctation commonest site


aortic coarctation....Common
classification
a- Infantile or preductal form
b- Adult or juxtaductal form
Adult or juxtaductal (postductal) form is more common, Usually localized , Area
of coarctation is just beyond the origin of LSCA at level of ductus.
6- Regarding thymus
a- irregular borders
b- should be seen at birth
c- should disappear by the age of one year
d- larger in erect that supine position in X ray
7- Most common site of pulmonary sequestration
a- Rt upper lobe
b- Lt upper lobe
c- Lt lower lobe
d- Rt lower lobe
8- image left lung multiple emphysematou bullae child with respiratory
distress(like this+pneumothorax
a- congenital lobar emphysema
b- emphysemtous malformation
c- ruptured bullae
d- adenatoid cystic mal-formation
9- Chest x ray in neonate showing left
mediastinal & tracheal shift with hyper
inflated right lung
a- left bronchial obstruction by foreign body
b- right congenital lobar emphysema
c- left bronchopneumonia
d- left lung collapse) lobar emphysema
10-copy past of this image congenital lobar
malformation

-87-

11- Child with dyspnea


there is 3 image 1st x
ray with rounded right
subdiaphragmatic opacity
2nd CT showing normal
lung with soft tissue
rounded opacity 3rd I
think celiac a angio
a- cystic fibrosis
b- round athelectasis
c- Hydatid cyst
12- Umbilical vein after labour converted to (ligamentum teres)
13- mcq about common causes of haemoperitoneum in neonate
a- adrenal haemorrahge
b- passage of blood from cord (frequent umblical cathter transfusions)
c- blood disease
d- NEC
N.B Common causes of neonatal hemoperitoneum ....breech/complicated
delivery, macrosomia,hepatoslpenomegaly,anoxic liver congestion. Direct and
indirect trauma during delivery to abdominal organs, liver, spleen, adrenals....
Neonatal Hemoperitoneum: Unexpected Birth Trauma with Fatal
Consequences.... http://www.ncbi.nlm.nih.gov/pmc/articles/PMC3830152/
14- Image plane x-ray for esophageal atresia
15- Mcq regarding congenital hypertrophic pyloric stenosis
a- it presents at 2 to 8 weeks of life
b- it presents with intestinal obstruction in x ray
c- x ray with contrast imaging plays minor rule in its diagnosis
d- US cannot differentiate stenosis from pyloric spasm.
16- Mcq Regarding reduction of intussuception in children;
a- pneumatic reduction is better than contrast reduction as there is less
exposure to radiation
b- it is contraindicated if symptoms persists for more than 24 hours
c- it is contraindicated if there is mechanical obstruction seen radiologically
-88-

d- it is contraindicated in recurrent cases


This method is quick and clean with a high reduction rate (73%95%) and less
radiation exposure than barium
enema therapy.The decreased
radiation exposure is related to
the shortened time required to
achieve reduction and the
milder (in terms of kilovolts
and milliamperes) radiographic
technique used) talking about
air enema

http://pubs.rsna.org/.../radiographics.19.2.g99mr14299
17- image ultrasound of hydrops fetalis
18- Regardoing DDH lines
a- Acetabular angle more than 30 suggest DDH
b- Hilgenreiner line is line vertical line drawn from the superior lateral rim of
the acetabulum
c- Perkins is Horizontal line connects the superior aspect of the triradiate
cartilage
19- DDH LINES AND ANGLES

-89-

a- Hilgenreiner line and Perkins line and Shenton line are useful in
diagnosis of congenital hip dislocatopn
b- other choices
DDH LINES AND ANGLES(definitions)
Hilgenreiner line is Horizontal line connects the superior aspect of the
triradiate cartilage bilaterally
Perkins line is A perpendicular vertical line drawn from the superior
lateral rim of the acetabulum through / corssing/ perpendicular to the line
of Hilgenreiner : Perkins line should intersect the medial femoral metaphysis
or the femoral head should project in the inferior medial quadrant created
by these lines (the femoral head usually ossifies by 2-3 months)
Shenton line is drawn along the undersurface of the femoral trochanter
and extends to the superior aspect of the obturator foramen Normally,
represents a contiguous arc (If noncontiguous, suspicious for DDH)
The angle of the acetabular roof can also be measured and should be less
than 30 degrees in the neonate and decrease as the hip matures (Diagnostic
imaging)

N<30

20- Mcq
the last
site of

Normal place of femoral


heads

ossification center at elbow joint appear


a- medial epichondyle
b- lateral epicondyle
c- troclea
d- capitulum

-90-

21- image a child does not eat vegetable what is the the diagnosis I write
scurvy
22- - Image : Rickets
23- Image : xray knee in patient with bleeding disorder
a- Hemophilia
b- thalathemia
c- rickets
24- About hip joint pathology in childeren
Hemophilia widening of the
A- Perth disease age between
interconylar notch,
6-9 months
accentuation of the
B- Sliped capital epiphysis
trabeculae, enlargment of
medial epicondyle
common between 6-8 years
25- 13 years old with delayed
bone age>>>>>
a- Scleroderma*
b- Crohn's disease---true
c- Hydatid disease
d- Gaucher disase
26- Regarding physiological
peri-osteal reaction in infant
a- may be irregular multilayered with bone thickening
b- involving mandible
c- occur in 35% of infant
d- may cause radioulnar synostosis
27- at what level the conus medullaris appears
a- L2-L3
b- L4-L5
c- L5-S1
d- L1-L2-----------adult
N.B Conus medullaris in children less than 3 months...L2-3, in adults L1-L2
28- about congenital lesions of the spinal cord
a- myelomeningiocel is associated with Chiari III.
b- Chiari II in almost 100% of cases.
b- Partial split is commoner than complete split in diastematomyelia (likely
true
29- Splitted cord image?diastomatomelia

-91-

30- regarding communicating hydrocephallus:


a- the temporal horn is the least to be dilated
b- the fourth ventricle is invariably dilated in all cases
c- CSF lumber puncture may show normal pressure = e.g.
NPH
31- MRI brain three images : sagittal with small cerebellum on apparently
large posterior fossa. Two axial cuts in high level at body of ventricle with
dilated ventricles and markedly reduced white matter with sulci reaching
deep to the ventricular wall. no interhemispheric fissure :
a- corpus callosum agensis
b- dandy walker syndrome
c- poly microgyra
32- image Fetal ultrasound:
a- Semilobar/alobar
holoprosencephaly.(true)

b- Hydranencephaly.
c- Anencephaly.
d- Hydrocephalus

33- Image MRI


in mutiple
sequencesAnd
history of
nephritic S come with distributed conscious level .
a- The answer is PRESS.
Posterior reversible encephalopathy syndrome http://radiopaedia.org/.../posterior-reversible...

-92-

34- regarding pediatric tumours


a- most common tumours are primary.
b- Three other options I dont remem
35- concerning children abuse :
a- brain injury is more common after 10 yrs
b- subarachnoid haemorrhage is uncomen below 10 yrs
c- brain ischemic changes are not common
d- injury not propionatal with history. True
The brain ischemic changes are common by either the edema or Hge.
CT findings in head trauma
Subdural hemorrhage (most common)
Interhemispheric location most common
Subarachnoid hemorrhage
Epidural hemorrhage (uncommon)
Cerebral edema (focal, multifocal, diffuse)
Acute cerebral contusion appears as ovoid collection of intraparenchymal blood with
surrounding edema
http://www.learningradiology.com/archives2008/COW%20335-Child
%20Abuse/abusecorrect.htm"Abuse/abusecorrect.htm
36- regarding sickle cell anemia:
a. CNS infarction is very rare
b. Moya moya arteriopathy appearance is seen.......... True
CNS infarction is very common in SCA. b is correct

-93-

L) Contrast and safety


1...7
14...less than 2 min
15...history for renal disease
1- mcq patient with triphasic CT has BP 80/50,HR 60/min, resp rate
12lLsec, what is the diagnosis
a- anaphlactic
b- vasovagal
vasovagal reaction :- managment : moniter vital signs, raise legs, secure
air way and IV access, push fluids.If pateint does not respond give atropine
0.6-1mg I.V slowly
N.B Adverse reactions to ICM are classified as idiosyncratic and
nonidiosyncratic
A- Idiosyncratic reactions typically begin within 20 minutes of the ICM
injection, independent of the dose that is administered. idiosyncratic reactions
to ICM are called anaphylactic reactions. The symptoms of anaphylactic
reaction can be classified as mild, moderate, and severe.
1- Mild symptoms include scattered urticaria, pruritus; rhinorrhea; nausea,
brief itching, and/or vomiting; diaphoresis; coughing; and dizziness No
TTT
2- Moderate symptoms include the following: persistent vomiting; diffuse
urticaria; headache; facial edema; laryngeal edema; mild bronchospasm or
dyspnea; palpitations, tachycardia, or bradycardia; hypertension; and
abdominal cramps-----need TTT.
3- Severe symptoms include the following: life-threatening arrhythmias (ie,
ventricular tachycardia), hypotension, overt bronchospasm, laryngeal
edema, pulmonary edema, seizures, syncope, and death.----need ICU.
B- Nonidiosyncratic reactions include the following: bradycardia,
hypotension, and vasovagal reactions;neuropathy; cardiovascular
reactions; extravasation; and delayed reactions
2- the most effective way to protect against toxic effect of ionic contrast is
a- Good hydration
b- Do not use contrast agent
c- alternative imaging studies e.g Ultrasound or non contrast mri
d- hemodialysis within 6 hours
N.B**** Patients with chronic renal insufficiency and a creatinine over 309.4
mol/L (3.5 mg.dl) who have elective coronary catheterization, a randomized
controlled trial found benefit from prophylactic hemodialysis.
3- The most effective way to prevent contrast induced nephropathy (CIN).
a-Good hydration within 18-24 hrs before exam
b- metformin cessation 48 hrs before
-94-

c- alternative imaging studies e.g Ultrasound or non contrast mri


d- hemodialysis within 6 hours
4- Nephrogenic systemic fibrosis does not occur if
a- does not occur if GFR more than 30
b- Immediate dialysis after IV contrast will prevent NSF
c- surgical intervention at the same time of gadolinum injaction exacerbate the
situation.
d- equal incidence in inpatient and outpatient
Many patients with nephrogenic systemic fibrosis have the triad of renal
impairment, a proinflammatory state, and gadolinium exposure . Most
patients have stage 4 (GFR less than 30 mL per minute per 1.73 m2) or
worse renal impairment, or are on hemodialysis or peritoneal dialysis, but two
case reports of nephrogenic systemic fibrosis in patients with a GFR greater
than 30 mL per minute per 1.73 m2 have been reported.11 Both cases involved
patients with acute renal failure, making the estimate of GFR less accurate
because it was based on a single serum creatinine level. Patients undergoing
peritoneal dialysis appear to be at the highest risk, with an incidence of 4.6
cases per 100 patients (versus 0.61 cases per 100 patients undergoing
hemodialysis).13 The increased risk may be due to poor clearance of
gadolinium by peritoneal dialysis or to a higher rate of MRI being ordered
for these patients in an attempt to avoid the nephrotoxicity of iodinated
intravenous computed tomography (CT) contrast media.14 The European
Society of Urogenital Radiology (ESUR) published guidelines in 2007 that
recommend against using gadolinium-based contrast agents in at-risk patients to
avoid the nephropathy associated with iodinated contrast media.15
5- The most important independent risk factor for contrast induced
nephropathy
a- Base line renal insufficiency (true)
b- DM
c- dehydration
d- hypertension
5- Increaes risk of nephrogenic systemic fibrosis
1- history of renal impirement"true
other answer were all wrong -2
http://www.radiologyinfo.org/en/safety/index.cfm...
Contrast-Induced Nephropathy
Patients with impaired kidney (renal) function should be given special
consideration before receiving iodine-based contrast materials by vein or

-95-

artery. Such patients are at risk for developing contrast-induced nephropathy, in


which the pre-existing kidney damage is worsened.
At-Risk Patients
Some conditions increase the risk of an allergic or adverse reaction to iodinebased contrast materials. These include:
previous adverse reactions to iodine-based contrast materials
history of asthma
history of allergy
heart disease
dehydration
sickle cell anemia, polycythemia and myeloma
renal disease
the use of medications such as Beta blockers, NSAIDs, interleukin 2
having received a large amount of contrast material within the past 24 hours
http://radiopaedia.org/articles/contrast-nephropathy
http://en.m.wikipedia.org/wiki/Contrast-induced_nephropathy
6- MCQ PT with bronchospasm after IV contrast what is the first line of
treatment
a- atropine
b- terbutaline ( I choose this as this is B2 agonist bronchodilatror)
c- antihistaminic
d- epinifrine
e- Albuterol
http://www.patient.co.uk/.../anaphylaxis-and-its-treatment
For mild bronchospasm, treatment includes oxygen 10-12 L by face mask,
close observation, and/or 2 puffs of an albuterol or metaproterenol inhaler.
For moderate cases without hypotension, treatment is as above, with
epinephrine 1:1000, 0.1-0.3 mL given subcutaneously, repeated every 10-15
minutes as needed until 1 mL is administered. Patients with severe
bronchospasm, administer epinephrine 1:10,000 1 mL slow intravenous
injection over approximately 5 minutes, repeated every 5-10 minutes as
needed. http://emedicine.medscape.com/article/422855-overview#aw
2aab
6b
9
7- Contraindication for the use of dipyridamole in stress perfusion imaging
of the heart
a- known case with 3 vessel coronary disease
b- aortic aneurysm
c- pt with peptic ulcer disease
d- PT has bronchospasm
-96-

Contraindications:
1. Asthmatic patients with ongoing wheezing should not undergo
dipyridamole stress test. However, it has been reported that patients with
controlled asthma can undergo the test and can have pre-treatment with two
puffs of albuterol or comparable inhaler.
2. Second or third degree AV block without a pacemaker or sick sinus
syndrome.
3. Systolic blood pressure <90 mmHg
4. Recent use of dipyridamole containing medications.
5. Known hypersensitivity to dipyridamole.
6. Unstable acute myocardial infarction or acute coronary syndrom
8-... ,, Nephrogenic Sclerosing Dermopathy
Contrast Induced Nephropathy Risk factors(image) Picture of inflamed ,
lower limbs in legs of two patients , the patient had a diagnostic study few
weeks ago what is mostly the study done:
a- Ionic contrast study.
b- Micro-bubble U/S.
c- Isotope study.
d- MRI with gadolinium. (true)

Photo of errythema of the hand another photo to the leg


Post Gd

9- to improve signal to noise ratio a reduction in which parameter would
influnce
a- slice thickness
b- magnetic filed strength
c- radiofrequency band width
N.B SNR can be improved by decreasing the noise ........by reducing the
band width, using surface coils and increasing number of excitations. AND
increasing the signal by decreasing the TE (time to echo) and increasing the TR
(time to repeat), slice thickness or field of view
10- Decrease of which parameter will decrease CT mottle

-97-

a-slice thickness
b- tube rotation time
c- tube KV
d- number of photons
Noise may be reduced by increasing the number of photons, by
increasing the slice thickness or pixel size. Noise can also be reduced by
increasing either MA or scan time
CT noise is generally reduced by increasing the kVp, mA, or scanning time
CT noise is also reduced by increasing voxel size, increasing reconstruction
field of view, increasing section thickness, or by image stacking
Noise can be reduced by using large voxels, increasing radiation dose, or using
a smoothing filter,.
11- to increase the photon energy you can increase
a- tube current
b- Tube voltage
c- target to film distance.
N.B my information is that KVp increases the energy of photons and tube
current (MAs increases the number of photon
12- CT pelvis with difference in CT number of both hips this could be duo
to
a- aliasing.
b- detector failure
c- increased fat
d- due to beam hardening artifact...
http://abcradiology.blogspot.com/2011/11/computed-tomography-artifacts.html
13- CT helical better than conventional in
A - spatial resolution---true
B- better contrast
C- less time
D- better storage data
14- In commerical CT scanners,the presetted perfusion time is
a- less than 2 minute.
b- 2-5 minute.
c- more than 5 minute.
For commercial CT scanners, scanning conditions for cerebral perfusion are
generally considered to be approximately 200 mAs and 3050-second scan time
(repeatable scans and intervals), giving total scan times of <25 seconds
precipitating factor for contrast renal failiure are -15
a-High dose gadolinium
b-Low dose gadolinium

-98-

c- history of renal disease


I went for renal disease
16- The only valuable procedure to reduce the harm effect of contrast
media is
a- vasodilator
b- hydration
c- sodium bicarbonate iv infusion
I choose hydration and I got full mark in safty and contrast module
Nephrogenic systemic fibrosis(NSF) usually happens with -17
a- Ionic Gadolinium
b- Non Ionic Gadolinium
c- High dose Gadolinium
d- Renal insufficiency
.Most probably d is the answer

-99-

M) Isotope
2...a
4...a
5...d
6 ..Tc Na pertech
1Image
isotope
scan
stress fracture
1st and 2nd
metatarsal bone

Stress fracture of 2nd


metatarsal bone

2- - Mcq bone scan time


a- during injection and immediatly and 4 hs :6 hs and 18:24 hrs
b- during and immedialy and 24 hs and 48 hs
c- during and immediately and 2 ds and 3 ds
http://radiopaedia.org/articles/bone-scan
3- Bone scan with progressive symmetrical uptake at lower femoral
condyles and humoral heads. No spleen uptake. there is an increasing
uptake of right femoral shaft as well as greater trochanter of the same bone
while left femur is free:
a- sickle cell anemia
b- diffuse metastases
c- osteoarthritis
4- hot uptake in a lesion within the right thyroid
a- toxic nodule
b- malignant nodule
c- residual after partial thyroidectomy
d- hashimoto thyroditis

-100-

131I , 123I or 99Tch scanning provides functional assessment of thyroid


Nodules classified as cold, warm or hot
Unable to differentiate benign and malignant nodules
Most solitary thyroid nodules are cold
Most cancers arise in cold nodules
Risk of cancer in a cold nodule is 10-15%
Risk of tumour in a hot nodule is negligible
Scintigraphy of minimal use in evaluation of solitary thyroid nodules
Useful in recurrent thyroid swellings and retrosternal goitres
5- Mcq,, GIT BLEEDING ISOTOPE STUDY patient underwent reb
blood cells tagged study for GI bleeding and the technician noted gastric
activity with free pertechinitate what would you do to confirm your
suspicious
a- Change patient position (make patient erect position)
b- Change size of photo beam( narrow photn beam)
c- Make patient drink 2 glasses of water
d- Scan thyroid
(dr osama saleh)
N.B in vivo labeling of the RBC's is not recommended as free technetium
will be excreted by the gastric mucosa and pass into the small bowel and colon.
Continuous gastric suction may be required if the in vivo labeling technique is
used. In vitro tagging results in significantly less amounts of free technetium.
Presently, the use of the ultratag kit for labeling the RBC's prior to GI bleeding
may reduce gastric activity.
6- Mcq,, agent used in MECKELS DIVERTICULUM isotope study...
99mTc-Na-pertechnetate
7- Image - Whole body scan with mediastinal active shadows bilaterally
rather symmetrical
a- lymph nodes
b- lymphoma
c- sarcoid

-101-

8- Image of V/Q scan of lung and degree of propality of pulm embolism.


High probability
two or more large mismatched segmental defects or equivalent
moderate/large defects with a normal x-ray
any perfusion defect substantially larger than radiographic abnormality
Intermediate probability
multiple perfusion defects with associated radiographic opacities
greater than 25% of a segment and less than two mismatched segmental
perfusion defects with:
normal radiograph
one moderate segmental
one large or two moderate segmental
one large and one moderate segmental
three moderate segmental
triple match: solitary moderate-large matching segmental defect with
matching radiograph
difficult to characterise as high probability or low probability
Low probability
Nonsegmental defects - small effusion, blunting costophrenic angle,
cardiomegaly, elevated diaphragm, ectatic aorta
any perfusion defect with substantially larger radiographic
abnormality
matched ventilation and perfusion defects with normal chest radiograph
small subsegmental perfusion defects
Normal scan
no perfusion defects

https://www.inkling.com/.../fundam.../chapter-55/figure-55-5
http://radiopaedia.org/articles/technetium-agents
9- mcq pt underwent cardiac scan during stress images there was septal
defect that refilled at rest images. yet the pt has no pain at the stress images
and the ECG was normal. so the cause of septal defect is:
a- myocardial ischemia
b- infarction
c- artifact
d- myocardial contusion

-102-

1)
-


(mcq) About hydatid disease choose the correct answer .
Casoni test is the serological test for diagnosis.(true)
The lung is the commonest site of affection.(false)
Most commonly present at 20 years of age. (F)
Aspiration is a diagnostic procedure (F)
Man is definite host ( F).

2) Regarding chondroid bone tumors


- Chondroblastoma affects immature skeleton.(true)
3)
-

(image) Elbow x-ray AP & lateral views :


Effusion with Fracture head of radius.(likely true)
Normal elbow
Elbow strain.
Effusion without fracture.

4) (image) Picture of CT chest anatomy 3 numbered structures were:


- Left brachiocephalic vein, Lt. C.C.A &pectoralis minor muscle.
(true)
- Left brachiocephalic vein, LT CCA & pectoralis major muscle.
5)
-

(mcq) About graves disease :


In thyroid ophthalmopathy pain is less than pseudotumor.
Deposition of glycoprotein at the tendinous insertion.
Severity of orbital disease correlates with clinical severity.

6) (mcq) CTA in pulmonary embolism:


- CTA is limited in subsegmentalpul. Embolism.(true)
7)
-

(mcq) Regarding seminoma the 1st lymph node group affected is :


Para-aortic.
Inguinal.
Along external iliac.
Internal iliac.

8) (image) Picture of renal scan , all of the following agents can be


used except:
- MAA ( mostly true)
- MAG3.
- I131 iodohippuran.
- Tc DTPA.

-103-

- Tc 99m EC.
9) (image) MRI pictures of hepatic focal lesion ( I dont remember the
signal changes of the lesion exactly but I think it was hypo in T1
hyper in T2 with post contrast enhancement).
- Haemangioma.(hypo, hyper peripheral nodular enhacement)
- FNH.(T1 iso to hypo (hypo scar), T2 iso to hyper (hyper scar)
Intense arterial, iso portal, scar delayed)
- HCC.(iso to hyper, variable to hyper, enh. Arterial, rapid wash, rim
enh)
- Cholangiocarcinoma.(capsular retraction, delayed persistant enh)
- Adenoma.(variable T1 mostly hyper, T2 mild hyper, enh art, iso
delayed)
10)
(mcq) The most common pattern of oesophageal atresia is
associated with (tricky one).
- Contrast study showing fistula between trachea and upper esophagus
(F)
- Gaseless abdomen (F)
- Trachea is non aerated due to congenital canalization defect (f)
- Lower lobe pneumonia is commonly found (According to
Danhert, upper lobe pneumonia).
- Xray chest and abdomen can be normal.
11) (mcq) The investigation of choice in patient with renal colic
having elevated renal creatinine.
- Unenhanced helical CT ( mostly true)
- Ultrasound.
- Isotope.
- IVP
12- (mcq) Regarding hypertrophic pyloric stenosis (tricky mcq).
- The aim of treatment is to relieve pyloric spasm by medical &
surgical treatment (true)
- congenital disorder affectting 2 -6 weeks of life. ( false). According to
danhert, acquired disorder not congenital.
- radiological imaging is the only diagnosis.
13- (image) CT images of Rt. Sided heart (dextrocardia / situs) the
question was This syndrome may be associated with all the following
except :
- Midline liver.
- Polysplenia.

-104-

- Rt sided stomach.
- Absent kidney. (likely the true one)
- interuption of IVC with continuation of azygous vein.
14- (mcq) About round atelectasis what is true:
- Exerts mass effect.
- Can show air bronchogram.(true)
- Ill defined in all margins.
- Pleural plaques never present with it.
15- (mcq) Contraindication of mammography :
- Pendulous breast.
- Bleeding per nipple.
- Recent surgical wound.
- Recent isotope scan.
16-

(mcq) Regarding benign ostegenic tumors:


Painful osteoid osteoma are seen on x-ray. (false)
In osteoblastoma, pain relieved by salicylates. ( false)
nidus > 2cm seen usually in osteoid osteoma. ( false)
recurrence after curratge or surgery in 60 to70%
Osteoblastoma - involvement of spine in 40% to 50%.(true)

17- Regarding large bowel;


- Most common cause of bleeding per rectum -Diverticulosis ( true).
- rectum is commonly involved in crhons disease. (false)
- caecum is most commonly involved by polyposis (False).
- adenomatous familial polyposis is Autosomal recessive disease
(False).
18- (mcq) CT of carotid artery dissection:
- Severe trauma is needed for dissection. (wrong)
- horner syndrome is a recognized association.(true)
- Recurrence at the same site.
- Appearance at angiography is cant be differentiated from
atherosclerotic disease.
19- Regarding enlargement of cardiac chambers on plain radiograph:
- The cardiac apex is displaced down and out with left ventricular
enlargement. (true)
- Enlargement of the left ventricle results in an elevated cardiac apex.
- Right atrial enlargement is a recognize cause of dysphagia.
- Right atrial enlargement is associated with splaying of carina.

-105-

- Left ventricular enlargement results in a decreased area of contact


between the front of heart and the sternum on lateral projection.
20- picture of mammography (ductography) with multiple filling
defects, this can be caused by all of the following except:
- air (gas bubbles).
- Multiple Papillomas.
- Granulomas.
- Invasive lobular carcinoma(True).
- blood clots
21- (mcq) Regarding bronchiectasis.
- Can occur temporary with pneumonia.(true)
- signet ring is rarely seen in HRCT. (false)
- congenital bronchiectasis occur with swayer james syndrome.
- Mucoid impaction is seen in all cases of bronchaiectasis.
22- Regarding thyroid cancer:
a) punctate / linear psammomatous calcifications at tumor
periphery on x-ray in papillary carcinoma (true).
b) papillary carcinoma shows the most hematogeneous spread.
c) Follicular carcinoma only shows lymph node involvement with rare
hematogeneous spread.
23- about mesothelioma. (chest CT):
a) associated with cigarette smoking; (F)
b) always associated with asbestos exposure. (F)
C) axial view of CT required to see diaphragm invasion. (F)
D) Ankylosing spondylitis causes thickening pleural apical cap. (f).
e) the last option is likely to be true which I do not remember.
(the pancreatic tumors doesnt cause pleural effusion)
24- Breast ultrasound.
a) U/S is investigation of choice in breast implant rupture.
b) U/S investigation of choice in patients age 50.
c) Surest sign of malignant axillary lymph nodes is loss of hilar fat.
D) Multiple intra-ductal papillomatosis shows malignant potential
(true).
e) intra-mammory lymph nodes needs serial follow up evaluaitons (false).
25- Regarding HRCT chest:
a) Sarcoidosis affects sub-pleural locations first (False).
- Pulmonary cystic changes are noted in lymphangiomyomatosis
affecting females.(true)
- Kerley B lines is the surest sign of acute left ventricular failure (False)
- Ankylosing spondylitis causes thickening pleural apical (False).

-106-

26) Chest X-ray:a)


Mitral valve calcification is seen in mostly with rheumatic fever
disease (l choose this).
27) (mcq) About Glioblastoma multiforms what is true:
- The least common tumor in the brain.
- minimal peri-lesional edema.
- associated with neurofibromatosis type 1 (TRUE).
- It involves mostly the gray matter.
- no contrast enhancement of tumor.
28) . (mcq) Most common cause of brain empayema:
- Sinusitis.(true)
- Otitis media.
- Postsurgical.
- Trauma
- Meningitis
29). Bone scan in adult patient presented by fever, Wt. loss, anorexia, in the
images, there is bilateral hilar rather symmetrical tracer uptake (no sex in
history)---what's the diagnosis:
a- lymphoma
b- sarcoidosis.
Lung abscess.
Metastases.
20
Wasan Alniaimi
cause of rt upper lobe consildation in acute M.I
bone scan ..sickle cell
the location of cardiac peacemaker in X-ray image: right atrium
CT chest
desequsmstive? or? cryptogrnic orgnized pneumonea??or pulmonary fibrosis??
i dont remmeber the fourth choice
female CT chest
asbestosis??..scleroderma??ipf?? I choosed scleroderma
CT chest...answer was lipoid pnuemonia
CT chest
kartenger??campbell syndrom??swyer james syndrom??eizenmenger
syndrom??
-107-

aortic angiography with atherosclerosis and one of the common illiacs


stenozed..asking about the best modality of treatment...answer is stent
If short stenosis the ttt is stent if long segment obstruction the ttt is surgical
bypass
thier was aquestion about minimal radiation dose for determenistic effect
...idont know the answer ..option was 0.2Gr, 2Gr, 200Gr
minimal radiation dose for deterministic effect for skin is 2-5 gy as in FARR
physics. so answer is 2
1.

2.

3.

4.

5.

6.


Lactating woman treated with radioative Iodin 131 about lactation
18-24 h
Continous lactation
Discontinue lactation
After one week
About cryptogenic pneumonia most specific
Air traping
Honey coombing
Invert halo sign
Centrilobular nodules
About galactocele
Appear fat in US
Rim of calcification in mammography
Fat in mammography
Aquastic shadowing in US
Most occult in angiography
Cavernous angioma
Venous anomoulus
AVM
AV fistula
Postpartum brain female had headach
Brain infarction
Lateral sinus thrombosis i choose this
Mets
If this means sheehan syndrome i think brain infarction
Lateral sinus thrombosis most common (hage or thrombosis)
(
Most pathgnomonic for tension pneumothorax
Bilateral
Invert diaphragm
Cardiomegally
Prominent hilar shadow

-108-

7. Angiography ECA aneurism which is affected


Visual
Mastication
Taste
Hearing
8. Female pt bloody diarrhia 3 month MRI small picture
Acute colitis
Chronic colitis i choose this
Toxic megacolon
9. MRI image axilal and sagital corpus callosal agenesis
Spot magnification
0.01mm
0.03mm
1mm
3mm
10. Dose related side effect of contrast media
Renal faluire (sure true i choose and get full mark)
bronchospasm
Laryngiospasm
Skin rash
11. Carotid doppler
I choose takayaso (thickinng of the wall)
Polyarteritis nodosa
Two other I dont remember
12. Aortic dissection affect more
Anterior part of descending
Posteror of descending
Posterio lateral part of descending
Lt. posterolateral in discending and antrolateral in ascending aorta
13. Anatomy of IAC or jagular foramin it was IAC by axial & cronal CT
14. CT chest:
hamartoma
15. Enlarged left kidney with round lesion at the pelvis take same contrast
of aorta
Aneurysm I choose this
Renal pelvis
Renal vein

US image for rt & lt kidney , sagittal MRI image for abdomen :
A) Rt hydronephrosis & Lt puj obstrction
B) Rt renal cystic , Lt puj obstructions
C) bilatral hydro nephrosis

-109-

Hamate

Ask about treatment


1- stenting
2-Endarterctomy
3-Surgical by pass
Surgical by
pass

metacarpal
Ask about
1- metastasis
2- osteosarcoma
3-Encondroma
Encondroma

1st



origin of the lesion
1- superior rectus ms
2- Lacrimal gland
3-Frontal sinus
5 5 lacrimal gland
head & neck

One question was in my exam: coronal MRI


angio of heart and great vessels, he was asking
about the name of this examination; my answer
which was wrong was ( post Gd enhanced
MRA) the correct answer is phase contrast
MRI, as the other two choices was wrong

-110-


1- inguinal H
2-spigelian hernia
3-internal H
spigelian hernia

MRI
Diagnosis
Appenditis
Colitis
Chron's
CHRONS

123

SMA
IMA

27/4/2016
Image show inverted halo sign

-111-

i think lipoid pneumonia.


Hilar mixed cystic lesion in female 20 years?
idiopathic inerstitial disease
bronchiectasis
pneumonia

bronchiectasis

Radiation dose of skin above which skin affected


2.20. 40 Gy, I think 2 Gy
TCC can affect
i think renal pelvis
renal vein, ureter
Mass related to liver and kidney but not attached to kidney and there is
line of cleavage with liver, hypodense in NCCT & enhanced in post
contrast with central hypodensity
The options were: adrenal or GB, HCC, RCC adrenal cancer
I think case showing right side aortic arch arrow point to the right side
arch
aortic arch mural non enhancing area and calcification at this area
in NCCT
acute dissection, mural hematoma,rupture aneurysm.
Hypodense no enhancing area surrounding left kidney
renal pancreatic origin, perinephric heamatoma
.hematoma
of digastric two anterior bellies ln
Submental
MRI spine in pt complaining of weakness: sagital T2 hypointense oval
intradural extramedullary lesion, enhanced on T1 with contrast:
,shwanoma, heamangioma, menigioma
MRI brain axial refer to ICA
CHPS, what is most diagnostic for it
Pyloric canal length >20mm
transverse diametre of pylous >13 mm,
Lytic bony lesion at base of metatarsal bone in patient aged 70
options no GCT ichosed osteosarcoma
Dilated bowel loop with inverted U shaped the dome of the U at Lt.
hypochondrium, the patient with history of constipation and old age:
-112-

I wrote it sigmoid volvulus but I found it cecal volvulus.



U shaped bowel loop with apex at Lt. hemidiaghragm
Cecal volvolus
Sigmoid volvolus
Not remembered Pt complaining of brownish urine, stone appeared in U/S
not in PUT, possible to be:
uric acid stone (true), other options Ca oxlate, struvite, ca phosphate.
sure sign of acute cholecystitis
(halo sign, peri GB fluid, wes sign, U/S murphy sign) I wrote peri GB fluid,
but it is U/S murphy sign.
Image of doppler showing what?
Aliasing
HRCT (bone alghorism, widow width-500, cardiac gating) regarding it.
widow level 700
Cystic fibrosis
a- chronic liver disease 70%
b- micro GB
c- calcification of pancreas 50%
Abdominal manifestations in cystic fibrosis are common, nearly all organ can
be affected. Only 39% of patients with cystic fibrosis (CF) have pulmonary
symptoms. 7% of CF patients do not present until adulthood.
Liver parenchymal disease
40% of CF patients will develop liver disease
1-8% progress to portal hypertension
40% progress to focal biliary cirrhosis (which is pathognomonic for CF)
Biliary tree
gallstones: 12-24%
sclerosing cholangitis
intrahepatic ductal strictures 100%
microgallbladder: 30%
Pancreatic manifestations
Exocrine insuffciency affects 85-90% Endocrine dysfunction occurs in 30-50%
of CF patients
fatty replacement
acute pancreatitis
pancreatic calcifications occur in 7%
pancreatic cysts: typically, microscopic 3mm diameter
pancreatic duct strictures

-113-

Gastrointestinal manifestations
gastro-oesophageal reflux & associated complications as Barrett
gastric & duodenal ulcers: likely related to impaired bicarbonate
secretion
distal intestinal obstruction syndrome
intussusception: typically ileocolic & may result from adherent fecal
residue or enlarged LN
pneumatosis intestinalis: confined to the colon
rectal mucosal prolapse
malignancies: of which colorectal carcinoma is most common
Patient with CT chest bilateral effusion more marked on Rt side with
enhancing nodules, the patient had history of cancer thyroid,
which is right mesothelioma or metastasis to pleura.
Image of ICA aneurysm, I think near its terminationm asking what is
affected
Hearing, vision, I wrote hearing.
Regarding to esophageal diverticulum which is true
a- Traction diverticulum is common at lower esophagus
b- Traction diverticulum occurs commonly in case of TB hilar lymphadenitis.
c- Postendoscopy is a common cause.
d- Multiple diverticulae can be seen at same time.

-114-

1-Proportion of lung hematoma for fat in CT (Fat % in hematoma)


5%, 15% , 35% ,75%
2-Glomus tumor and shwannoma in MRI
3-Exstraperitoneal ruptuer of urinary bladder in US
4-12y boy with sever reccurent epistaxis , vascular mass appear in
nasopharynx on ct .treatment by :
Embolization before surgury
Endoscopic removal
Laryngoscopic removal
Other choice
5- Adernal adenoma in CT
unenhanced less than 10 HU
enhanced less than 45 HU

6- Sudden central abd. pain in male There is small image (angio) of dilated
vessels ( gastic varices) he asked about ttt:
Glue, ingection of thrombolitic, balloon
if the case is gastric varices with acute bleeding then the standard tx is Glue
injection through endoscopy
7- fibroadenoma in US:
Fat density, hypoechoic
8- subacute hypersensitivty pneumonitis specific feature in ct:
Centerlobular nodule, air trapping
9- Female with shoulder pain and +ve neers test (MRI image):
Acromion impingement, supraspinatus injury,
10- male presented with dysphagia & shoulder pain what is the diagnosis
(2 CT images):
Oesophageal cancer, lung ca
The image like this without invasion of trachea
Dose of skin exposure (0.2 - 2 - 20 - 200 Gy)
Carotid Doppler name of artifact (aliasing)
Amiodrone toxicity in chest radiograph
peripheral areas of consolidation with
upper lobe predominance (most
common) +/- underlying interstitial disease; interstitial fibrosis.

-115-

Renal scan tc99m tracer best to evaluate


glomerulonephritis- tubular necrosis- vesicoureteric reflux- renal vascular HTN
2 images of coronal CECT chest old male with chest pain
acute dissection - mural hematoma - other
Aerobilia post cholecystectomy
post ERCP Sequlae
post cholecystectomy ERCP images
intraductal calculi
20y male diarrhea, loss of weight 2 MRI coronal images very bad quality
adenocarcinoma-lymphoma
2 MRI
bilateral adrenal lipid rich adenoma - bilateral pheochromocytoma
Corpus callosal agenesis MRI sag & axial images
Regarding HRCT
window level - width - +/- 500 - bone algorithm - cardiac monitoring

Ependymoma axial & sagittal MRI like this image

X lnked leukodystrophy like this


-116-

Image like this pointing to coronary sinus

Image like this pointing to left renal vein

The most common brain tumor in pediatrics is ......primary CNS tumor


Neuroblastoma
a- calcification in 80 % in X-ray
b- extention into spinal canal (b is sure answer)
-117-

Chairi II characterized by myelomeningocele


Axial Ct Brain show the following intraparenchymal , intraventricular,
subarachnoid , extracranial , midline shift and effaced sulci ....take care
befor answer as choices are similar but all this findings can be seen
Female Pt presented by severe headache to Emergency and CT revealed
subarachnoid what is the next widely used investigation
a- CTA
b - conventional angio c- MRI
(a is sure answer to exclude aneurysm)
Focal spot size (mm) in new advanced mammo 0.01, 0.03, 1, 3 0.03
The focal spot sizes commonly employed are: 0.3mm & 0.6mm, usually for
mammo 1.0mm & 1.2mm, usually for general radiographyas in Radiopedia
breast full mark
a- .01 mm
B- .03 mm
C- .10 mm
D- .30 mm
d c or d

Female with connective tissue disease complain of limb weakness....MRI
image bright signal in T2 fat sat, I see T1 normal
Rhabdomyolysis, myositis, atrophy, myonecrosis
myositis
I think also it is MYOSITIS which is associated with connective tissue disease,
atrophy will be replaced by fat so hyper in T1W , Rhabdomyolysis is related to
drugs and myonecrosis related to DM and there is collection
Expansile lytic lesion in the distal radius in male 60 y with extraosseous
soft tissue
GCT
ABC
Primary lymphoma
GCT may be locally agressive 10% Primary lymphoma is very rare
Ureteric diversion what is the early complication
Extravasion
Chronic pyleonephritis
Ureteric stenosis
extravasation GIT problems is early complication
Which structure in elbow liable to valugus trauma
Familial polyposis & CNS tumours
turcot syndrome, la framini, cowden, tuberos sclerosis. .. turcot
-118-

Neonate with liver cirrhosis small gall bladder , not visualized spleen
Physiological jaundice
Neonatal hepatitis
Biliary hypoplesia
Biliary atresia
Anterior mediastinal mass
Lymphoma
Teratoma
Ct chest with embolus on right main pulmonary artery ,but the lesion at
left lung subpleural round consolidation and mild left pleural effusion

-119-

Pulmonary infarction
Hamartoma
Atelactesis
Meningioma
Shawnnoma
Adenoid cystic carcinoma
Stenosis in celiac trunk

isotope
I think bilateral hilar uptake Sarcoidosis
Osteo meatbal unit drain the following
sinus
Frontal, maxillary, anterior ethmoid
Anatomy MRI of heart coronal at the level of ventricles
Coronary sinus
Pericardial recess
Right pulmonary artery
Right superior pulmonary vein
MRI abdomen of normal sized colon for female Pt complaining of bloody
diarrhea for 3months
Acute colitis
Chronic colitis
Acute toxic colitis
Hirschsbrung

-120-

CT intussception
Emphysemtous pyelonephritis
Post op. hip prosthesis with X-ray abdomen I did not see fluid level but
horizontal dilated bowel at the pelvis
Bowel perforation
Paralytic ilius
Intestinal obstruction
Not rembered
Which characteristic for aortic hematoma in CECT
Hyperdense in CT
Peri aortic soft tissue
Mural flap

Which ch ch for empyema
Split pleura
Lt. renal mass in CT with renal & splenic v. invasion further scanning for
mets:
PET CT
CT chest
MRI chest
oncology imaging book say that PET is less sensitive than CT in general mets
of RCC however in bone mets is more sensitive than CT.
IVU image with Filling defect in UB
Schestisomiasis
Cancer bladder
Renal cancer
CT chest aspergilloma I think in patient known to have tuberculosis

GCT
ABC
Primary bone lymphoma

CXR I can't recognize any abnormality
HRCT showed few cystic bronchiectasis, no
cardiac nor lung annormality
kartenger
Campbell syndrome
swyer james syndrome
eizenmenger syndrome
-121-

CXR showed Rt upper lung opacity, history of Lt ventricular infarction Q


is about cause of this opacity
a. lung contusion
b. lung hematoma
c. neurogenic oedema
d. papillary ms rupture...
Benign looking lesion in mammo with periphral areas of low attenuatio &
central dense area:
A.oil cyst
B.fibroadenolipoma
C.intraductal carcinoma.
D.phyloid tumor
CT chest showing plueral calcified plaques with plueraly based nodule:
asbestosis
Rt sided aortic arch % in fallots 4:
50%
25%
75%
10%
CT chest: posterior mediastinal small cyst, in CECT no enhancement:
Duplication cyst
Male Pt going to do knee replacement, CECT chest showed small Rt upper
lobe nodule showing pop corn calcification difficult to detect calcification
in postcontrast CT
Hamartoma
CT chest with interstitial changes. The image was lung window only I think
lower lobe I think oesophegus was mildly dilated:
A. IPF
B. scleroderma
C. sarcoidosis
D. asbestosis.
CT chest: mosiec pattern of attenuation:
A. ch.pulmonary embolism.
B.pulm oedema
C.pneumonia
:
A.lymphoma
B.sarcoidosis
C.pulm oedema
D.Lt to Rt shunt
it was pulmonary a. enlargement

-122-

:
A. Douple IVC
B. Renal agenisis
C. Lt renal thrombosis
D. Lt common iliac v thrombosis
It was a IVC venogram with Lt CIV thrombosis
Small cell lung car mets to brain which is
more spicific:
A. vasogenic oedema
B. high T2 signal
C. hemorrhage
D. no enhancement
CT without contrast: Lt ovary showed small
cystic mass lesion with small focus of
calcification & fat
Teratoma
MCQ: cystic lesion in femal pelvis
showing fluid fat level in CT
Dermoid cyst
Picture similer to this with areas
showing douple wall HRCT slice
thickness
MCQ: regarding meningioma:
A.hyper T1
B.hypo T2
C.intraaxial
D.no contrast enhancement (I choose b as iso/hypo T1 & iso/hypo/hyper T2)

Femur with thick cortix + isotope I think e
superscan (17 years old, bilateral
symmetrical cortical sclerotic disease, no
bone expansion or altered trabeculation):
A.diaphysial dysplasia
B.paget disease
C.lymphoma
my answer was diaphyseal dysplasia.

-123-

Вам также может понравиться